You are on page 1of 64

Chapter

8 Newton's Laws of
Motion
Types of forces |F| ∝ x and F = −kx, where F = spring force, x = compression or
zz Contact forces : If two surfaces are in physical contact extension and k is spring constant.
(touching each other), contact forces come into picture.
Tension
The component of the contact force : normal reaction and
Tension (usually denoted by
friction.
T) is always a pulling force.
zz Non-contact forces : Without actual physical contact,
It can never push a body.
bodies can exert forces on one another. Examples are
If the string is massless, then tension throughout the string is
gravitational force, electrostatic force, magnetic force etc
same. If the string has (considerable) mass, tension at different
Weight points in it will be different.
Weight of a body is the force with which it is attracted by the
earth. Its direction is always downwards (i.e., towards the Newton’s first law of motion (or law of
centre of the earth). inertia)
The net force on a body which is at rest or in uniform motion
Normal force or normal reaction is zero.
It always acts normal to the surface of contact (or line of zz Inertia of rest : The inability of a body which is at rest, to
contact). change its state of rest on its own i.e., without an external
force, is known as inertia of rest.
zz Inertia of motion : The inability of a body which is in
uniform motion, to change its state of motion on its own
i.e., without an external force is known as inertia of
motion.
zz Inertia of direction : The inability of a body to change its
Normal reaction(N) on block is along CA direction of motion on its own is called inertia of direction.
Normal reaction(N) on inclined plane
Normal reaction on A is upwards
Normal reaction on B is downwards

is along CB
Linear momentum ( p)
Spring force The product of mass and velocity of a body is defined as its

It is proportional to the extension (or compression), but opposite linear momentum ( p ). Sometimes it is simply called
to the extension (or compression). momentum.
  
p = m v , where m = mass of the body, and v = velocity
zz

of the body
zz Momentum is a vector physical quantity. Its direction is
same as that of velocity. Its SI unit is kg ms−1 (or Ns)
p 2 , where K is kinetic energy of
=
zz p = 2 Km and K
2m
the particle.

Newton’s second law of motion


zz It states that the rate of change of momentum of a body is
directly proportional to the external force acting on it and
takes place in the direction of force.
8.2  Physics

  Points to be remembered while drawing FBD’s


dp   dv  
∴ ∝ F or F = m ⇒ F = ma zz Represent the weight of the body.
dt dt 
 dv zz If connected to a string, represent tension. Note that it
( a is acceleration equal to )
dt always has pulling effect.
zz The SI unit of force is Newton (N) and its CGS unit is zz If connected to a spring, represent spring force. If the
dyne. spring it extended it has pulling effect. If it is compressed
1 N = 105 dyne. The gravitational unit of force is kg wt or it has pushing effect.
kg(f) [kilogram weight or kilogram force] zz If the body is in contact with a surface, represent normal
1 kg wt = 1 kg(f) = 9.8 N on earth. reaction.
zz The slope of p−t (momentum − time) graph gives the (The other contact force is frictional force and will
force.
be considered later). Note that the normal force is
perpendicular to the surface of contact (or line of contact).
zz If there are any applied forces, represent them.

If a pulley is massless or light, its weight is not


 drawn and the net force on it is zero. [ F = ma and
mass of the pulley, m = 0]
The net external force at time t1 = tan θ1
 at time t2 = tan θ2
The net external force Problem solving strategy – applying Newton’s
zz The direction of F is along a (or) along change in laws

momentum (D p) of the body. zz Decide the system: The system, onto which laws of
motion are to be applied, is to be identified. If the system
Law of conservation of linear momentum
is not a single body, but a collection of two or more
In case the external force applied to a particle (or a body) is
zero, then bodies, the only condition is that all the bodies must have
 dp  same acceleration.
F= = 0 ⇒ p = constant zz Note down the forces acting ON the system
dt 
In the absence of an external force (or Fext = 0) the linear zz Draw the FBD of the system
momentum of a particle (or a body) remains constant. This law zz Choose the axes and write the equations: If the forces
is called the law of conservation of linear momentum. are coplanar X and Y axes are chosen. The forces are
 resolved along X and Y axes. Then we have two equations.
Impulse ( J )
 ∑Fx = max and ∑Fy = may
If a large force ( F ) acts on a body for a short interval of time
If the system is moving along X-axis, then ay = 0.
(Dt), it is called impulsive force. The product of this impulsive
force for which it acts is called impulse. ∴ ∑Fy = 0
 and the time   If the forces are collinear, we do not require the equation
J = F ∆ t or J = ∆p
∑Fy = 0.
 t2 
Also, J = ∫ F dt , when force is a function of time
Lami’s theorem
t1
zz Impulse is a vector. The direction of impulse is along the It states that if three coplanar forces acting on
force or change in momentum. a particle keep it in equilibrium, then
zz Its SI unit is Ns (or) kg ms−1 P Q R
= =
zz The area under F − t (force − time) graph gives impulse. sin α sin β sin γ
Newton’s third law of motion Where P, Q and R are magnitudes of forces
It states that to every action there is equal and opposite reaction. and α, β and γ are the angles between them as
Suppose that a body A experiences a force FAB due to a body shown in the figure.
Here the equilibrium means static equilibrium (i.e., the body
B. Also body
 B will experience a force FBA due to A, then
is stationary) or dynamic equilibrium (i.e., the body is moving
FAB = − FBA or Action = −Reaction
with constant velocity).
Action and reaction never cancel each other because they act on
two different bodies. Inertial and non-inertial frames of reference
Free body diagrams (FBD’s) A reference frame at rest or moving with constant velocity
Free body diagram of a body (or a system) gives all the forces with respect to another inertial frame is called inertial frame of
acting ON the body (or the system), with magnitudes and reference. A reference frame that is accelerating is called non-
directions. inertial frame of reference.
Newton's Laws of Motion  8.3

Important points
Elevator : Non-inertial frame zz (a) If a body of mass m, is in an elevator accelerating
upwards, its apparent weight is N = m(g + a).
(b) If the same body is in the elevator decelerating
upwards, its apparent weight is N = m(g − a)
zz (a) If the same body is in the elevator accelerating
downwards, its apparent weight is N = m(g − a).
(b) If the same body is in the elevator decelerating
downwards, its apparent weight is N = m(g + a).
zz If a block is suspended by a massless string from the
Ground : Inertial frame ceiling of the elevator, in the above cases, N is replaced
by tension (T) in the string.
FBD of m wrt A (Inertial frame) zz Pseudo force on a body of mass m is to be applied in a
direction opposite to the acceleration of the non-inertial
Fnet = N − mg …. (upwards)
frame.
∴ N − mg = ma
or N = m(g + a) Motion of a body on a smooth inclined plane
The forces the given body mass
FBD of m wrt B (Non-inertial frame) in it are its weight (mg) vertically
Now, N = mg + ma downwards and normal reaction
or, N = m(g + a), which is same as obtained (N) perpendicular to the
by observer A. inclined plane. If mg is resolved
perpendicular and parallel to
So, it is seen that whenever a problem
the inclined plane, we get
is solved from a non-inertial frame of mg sin α = ma ⇒ a = g sin α (where a is acceleration of body
reference, pseudo force has to be included  down the plane)
in the FBD of the body analyzed. N = mg cos α
Newton’s Laws of Motion

First Law of Motion ward just as hard as the locomotive pulls forward,
there is no motion.
1. A boy sitting on the topmost berth in the compartment
(c) The locomotive’s force on the wagons is as strong
of a train which is just going to stop on a railway station,
as the force of the wagons on the locomotive, but
drops an apple aiming at the open hand of his brother
the frictional force on the locomotive is forward
sitting vertically below his hands at a distance of about
and large while the backward frictional force on the
2 meter. The apple will fall
wagons is small.
(a) Precisely on the hand of his brother
(d) The locomotive can pull the wagons forward only if
(b) Slightly away from the hand of his brother in the
it weighs more than the wagons.
direction of motion of the train
(c) Slightly away from the hand of his brother in the 5. What net force is required to keep a 500 kg object
direction opposite to the direction of motion of the moving with a constant velocity of 10 m/s?
train
(d) None of the above Second Law of Motion
2. A person sitting in an open car moving at constant 6. In the figure given below, the position-time graph of a
velocity throws a ball vertically up into air. The ball particle of mass 0.1 kg is shown. The impulse at t = 2 sec is
falls
m
(a) Outside the car
(b) In the car ahead of the person
(c) In the car to the side of the person
(d) Exactly in the hand which threw it up
3. A mass of 1 kg is suspended by a string A. Another
string C is connected to its lower end (see figure). If the
string C is stretched slowly, then
(a) 0.2 kg m sec–1 (b) –0.2 kg m sec–1
–1
(c) 0.1 kg m sec (d) –0.4 kg m sec–1
7. A body of mass 2 kg has an initial velocity of 3 meters
per second along OE and it is subjected to a force of 4 N
in a direction perpendicular to OE. The distance of the
body from O after 4 seconds will be
(a) The portion AB of the string will break (a) 12 m (b) 20 m
(b) The portion BC of the string will break (c) 8 m (d) 48 m
(c) None of the strings will break 8. Which of the following conclusion is correct regarding
(d) None of the above a stationary body?
4. A locomotive (engine) pulls a series of wagons. Which (a) No force is acting on the body.
is the correct analysis of the situation? (b) Vector sum of forces acting on the body is zero.
(a) The train moves forward because the locomotive (c) The body is in vacuum.
pulls forward slightly harder on the wagons than the (d) The forces acting on the body, do not constitute a
wagons pull backward on the locomotive. couple.
(b) Because action always equals reaction, the locomo- 9. When forces F1, F2, F3 are acting on a particle of mass
tive cannot pull the wagons. The wagons pull back- m such that F2 and F3 are mutually perpendicular, then
2 Physics

the particle remains stationary. If the force F1 is now F(N)


removed then the acceleration of the particle is
(a) F1/m (b) F2F3/mF1 (III)
1.0
(c) (F2 – F3)/m (d) F2/m
     t(10–3s)
10. Five forces F1 , F2 , F3 , F4 and F5 are acting on a particle 0 1.0
of mass 2.0 kg so that it is moving with 4 m/s2 in east F(N)

direction. If F1 force is removed, then the acceleration
becomes 7 m/s2 in north, then the acceleration of the (IV)
1.0

block if only F1 is acting will be:
t(10–3s)
(a) 16 m/s2 (b) 65 m/s2 0 1.0

(c) 260 m/s2 (d) 33 m/s2 The impulse is highest in the case of situations depicted
in the figures.
11. The force–time (F–t) curve of a particle executing
(a) I and II (b) III and I
linear motion is as shown in the figure. The momentum
(c) III and IV (d) IV only
acquired by the particle in time interval from zero to 8
second will be 14. A 2 kg toy car can move along an x-axis. Graph shows
force Fx, acting on the car which begins at rest at time
+2 t = 0. The velocity of the particle at t = 10 s is:
Force (N)

2 4 6 8 Fx(N)
4
Time (s)
–2

(a) – 2 N-s (b) + 4 N-s


(c) 6 N-s (d) Zero t(s)
12. A force–time graph for the motion of a body is shown 0
4 8 9 10 11
in the figure. The change in the momentum of the body
between zero and 10 sec is –2

1 (a) – iˆ m/s (b) – 1.5 iˆ m/s


(c) 6.5 iˆ m/s (d) 13 iˆ m/s
F
15. A particle of mass m and initially at rest is acted by a
(N)
10 t(s) x
force F = at newtons. Best representation of force–
8
0.5 displacement graph is:
F F
(a) zero (b) 4 kg m/s
(c) 5 kg m/s (d) 3 kg m/s (a) (b)
S S
13. Figures I, II, III and IV depict variation of force with time
F(N) F F

(c) (d)
S S
(I)
0.25

t(10–3s) 16. A 15 kg block is initially moving along a smooth


0 1.0 horizontal surface with a speed of v = 4 m/s to the left. It
is acted by a force F, which varies in the manner shown.
F(N)
Determine the velocity of the block at t = 15 seconds.
F(N)
0.3
(II) v
F 40
t(10–3s) 10
5 15
0 1.0 2.0 t(s)
Newton’s Laws of Motion 3

Êpˆ 24. A ball of mass m falls vertically to the ground from a



Given that, F = 40 cos Á ˜ t height h1 and rebound to a height h2. The change in
Ë 10 ¯
momentum of the ball on striking the ground is
(a) 12.5 m/s (b) 8.5 m/s
(a) mg(h – h ) (b) m( 2 gh1 + 2 gh2 )
(c) 20 m/s (d) 9.5 m/s 1 2

17. A solid sphere of mass 2 kg is resting inside a cube as m 2 g (h1 + h2 )


m 2 g(h1 + h2 ) (d)
(c)
shown in the figure. The cube is moving with a velocity
25. A particle of mass M is moving in a horizontal circle of
v = (5t iˆ + 2t ˆj ) m/s. Here t is the time in second. All
radius R with uniform speed V. When it moves from one
surface are smooth. The sphere is at rest with respect to
point to a diametrically opposite point, its
the cube. What is the total force exerted by the sphere
on the cube. (Take g = 10 m/s2) (a) kinetic energy changes by MV2/4
(b) momentum does not change
(c) momentum changes by 2 MV
(d) kinetic energy changes by MV2
26. A particle moves in the X–Y plane under the
influence of a force such that its linear momentum

is p(t ) = A[iˆ cos (kt ) - ˆj sin (kt )], where A and k
(a) 29 N (b) 29 N are constants. The angle between the force and the
(c) 26 N (d) 89 N momentum is
18. Two masses of 1 g and 4 g are moving with equal (a) 0° (b) 30°
kinetic energies. The ratio of the magnitudes of the (c) 45° (d) 90°
linear momenta is 27. A particle moves in the xy-plane under the action of a
(a) 4 : 1 (b) 2 :1 force F such that the components of its linear momentum
(c) 1 : 2 (d) 1 : 16 p at any time t are px = 2 cos t, py = 2 sin t. The angle (in
degree) between F and p at time t is
19. Two bodies with kinetic energies in the ratio of 4:1 are
moving with equal linear momentum. The ratio of their 28. If a bullet of mass 5 gm moving with velocity 100
masses is m/sec, penetrates the wooden block up to 6 cm. Then
(a) 1 : 2 (b) 1 : 1 the average force (in N) imposed by the bullet on the
(c) 4 : 1 (d) 1 : 4 block is
29. A ball of mass 400 gm is dropped from a height of 5 m. A
20. If the K.E. of a body is increased by 300%, its momentum
boy on the ground hits the ball vertically upwards with a
will increase by
bat with an average force of 100 newton so that it attains
(a) 100% (b) 150%
a vertical height of 20 m. The time (in s) for which the
(c) 300% (d) 175% ball remains in contact with the bat is [g = 10 m/s2]
21. If the kinetic energy of a body increases by 0.1% the 30. A 100 g iron ball having velocity 10 m/s collides with a
percent increase of its momentum will be wall at an angle 30° and rebounds with the same angle.
(a) 0.05% (b) 0.1% If the period of contact between the ball and wall is 0.1
(c) 1.0% (d) 10% second, then the force (in N) experienced by the wall is
22. A body of mass 0.5 kg is projected under the gravity 31. A bullet is fired from a gun. The force on the bullet is
with a speed of 98 m/s at an angle of 30° with the given by F = 600 – 2 × 105 t, where F is in newtons and
horizontal. The change in momentum (in magnitude) of t in seconds. The force on the bullet becomes zero as
the body when it strikes the ground is soon as it leaves the barrel. What is the average impulse
(a) 98 N–s (b) 49 N–s (in Ns) imparted to the bullet
(c) 196 N–s (d) 24.5 N–s 32. At a certain instant of time the mass of a rocket going
23. A particle of mass m is executing uniform circular up vertically is 100 kg. If it is ejecting 5 kg of gas per
motion on a path of radius r. If p is the magnitude of second at a speed of 400 m/s, the acceleration (in m/s2)
its linear momentum, then the radial force acting on the of the rocket would be (taking g = 10 m/s2)
particle is 33. Five persons A, B, C, D and E are pulling a cart of mass
(a) p m r (b) r m/p 100 kg on a smooth surface and cart is moving with
(c) 2
m p /r (d) p2/r m acceleration 3 m/s2 in east direction. When person A
4 Physics

stops pulling, it moves with acceleration 1 m/s2 in the 2


west direction. When person B stops pulling, it moves 1 Ê Lˆ
(iv) The total K.E. of the system is (m + M ) Á ˜ .
with acceleration 24 m/s2 in the north direction. The 2 Ë t0 ¯
magnitude of acceleration (in m/s2) of the cart when
(a) (i), (ii) (b) (i), (iii)
only A and B pull the cart keeping their directions same
(c) (i), (iii), (iv) (d) (i), (ii), (iv)
as the old directions, is:
34. The given figure shows a bead of mass m moving with 38. Consider the following two statements:
uniform speed v through a U-shaped smooth wire. The 1. Linear momentum of a system of particles is zero
wire has a semicircular bending between A and B. The 2. Kinetic energy of a system of particles is zero, Then
average force exerted by the bead on the part AB of the (a) 1 implies 2 and 2 implies 1
kmv 2 (b) 1 does not imply 2 and 2 does not imply 1
wire is . Find k.
πd (c) 1 implies 2 but 2 does not imply1
v (d) 1 does not imply 2 but 2 implies 1
A m 39. A shell of mass m moving with velocity v suddenly
d breaks into 2 pieces. The part having mass m/4 remains
B
stationary. The velocity of the other shell will be
(a) v (b) 2v
35. A particle of mass 500 g is projected along x-axis
3 4
with a velocity of 6 m/s. It is acted upon by variable (c) v (d) v
force acting along y-axis as shown in the figure. The 4 3
modulus of velocity (in m/s) at t = 8 s is 10n. Find 40. A stationary particle explodes into two particles of
the value of n. masses m1 and m2 which move in opposite directions
F with velocities v1 and v2. The ratio of their kinetic
1N
energies E1/E2 is
(a) m1/m2 (b) 1
t
(c) m1v2/m2v1 (d) m2/m1
4s 8s 12 s 16 s
     41. A body of mass 50 kg is projected vertically upward
36. Five forces F1 , F2 , F3 , F4 and F5 are acting on a with velocity of 100 m/s. After 5 s this body breaks into
particle of mass 2.0 kg so that it is moving with 4 m/ 20 kg and 30 kg. If the 20 kg piece travels upwards with

s2 in east direction. If F1 force is removed, then the 150 m/s, then the velocity of other block will be
acceleration of the particle becomes 7 m/s2 in north. If (a) 15 m/s downwards

the acceleration of the particle is n m/s2 if only F1 is (b) 15 m/s upward
acting, then find n. (c) 51 m/s downwards
(d) 50/3 m/s downwards
Conservation of Momentum 42. A shell is fired from a cannon with velocity v m/s at
37. A wooden plank of mass M and length L is floating in an angle q with the horizontal direction. At the highest
still water. A person of mass m starts at one end of the point in its path it explodes into two pieces of equal
plank and reaches to other end in time t0, moving with a mass. One of the pieces retraces its path to the cannon
constant speed. Choose the correct option. and the speed in m/s of the other piece immediately
after the explosion is
(i) The speed of the person as seen from the ground is
L (a) 3v cos q (b) 2v cos q
smaller than .
t0 3 3
(c) v cos q (d) v cos q
(ii) The speed of the plank as seen from the ground is 2 2
Ê m ˆ L 43. At high altitude, a body explodes at rest into two equal
. fragments with one fragment receiving horizontal
ËÁ m + M ¯˜ t0
velocity of 10 m/s. Time taken by the two radius vectors
(iii) The speed of the plank as seen from the ground is connecting point of explosion to fragments to make 90° is
Ê M ˆ L (a) 10 s (b) 4 s
ÁË m + M ˜¯ t . (c) 2 s (d) 1 s
0
Newton’s Laws of Motion 5

44. A bomb explodes in air when it has a horizontal speed One of them moves horizontally with speed 20 m/s. The
of 10 m/s. It breaks into two pieces A and B of mass ratio distance between the two pieces on the ground is
1:2. If A goes vertically up at a speed of 40 m/s, the (a) 20 m (b) 40 m
speed of B is (c) 60 m (d) 80 m
(a) 20 m/s (b) 25 m/s
50. A cannon shell is fired to hit a target at a horizontal
(c) 30 m/s (d) 50 m/s
distance d. However, it breaks into two parts of mass
45. A body at rest breaks up into 3 parts. If 2 parts having ratio 1 : 2 at its highest point. The smaller part returns to
equal masses fly off perpendicularly each after with a cannon. The other part
velocity of 12 m/s then the velocity of the third part (i) will fall at a distance d/2 beyond the target
which has 3 times mass of each part is (ii) have eight times the kinetic energy of smaller part
(a)
4 2 m/s at an angle of 45° from each body (iii) the increase in kinetic energy of system after explo-
sion is 200%
(b)
24 2 m/s at an angle of 135° from each body (iv) after explosion, heavier part will take more time as
(c)
6 2 m/s at 135° from each body compared to smaller part, to strike the ground
(a) (i), (ii) (b) (i), (ii), (iii)
(d)
4 2 m/s at 135° from each body (c) (i), (iii) (d) all options are correct
46. An explosion blows a rock into three parts. Two parts 51. A man is standing at the center of frictionless pond of
go off at right angles to each other. These two are 1 kg ice. How can he get himself to the shore?
first part moving with a velocity of 12 m/s and 2 kg
(a) By throwing his shirt in vertically upward direction
second part moving with a velocity of 8 m/s. If the third
(b) By spitting horizontally
part flies off with a velocity of 4 m/s, its mass would be
(c) He will wait for the ice to melt in pond
(a) 5 kg (b) 7 kg
(d) Unable to get at the shore
(c) 17 kg (d) 3 kg
52. A bullet is fired from a rifle. If the rifle recoils freely,
47. An object, initially at rest, explodes in three fragments. then the kinetic energy of the rifle is
The momentum of two pieces are -3p iˆ and -4 p ˆj
(a) less than that of the bullet
where p is a positive number. The momentum of the
(b) more than that of the bullet
third piece
(c) same as that of the bullet
(i) will have magnitude 5p (d) equal or less than that of the bullet
(ii) will make an angle tan–1 (4/3) with the x-axis
53. A boy of mass m is standing on a platform of M kept on
(iii) will make an angle tan–1 (3/4) with the x-axis
smooth floor. If the boy starts moving on the platform
(iv) will have magnitude 7 p with a speed v0 relative to the platform, with what
(a) (i), (iii) (b) (i), (ii) velocity relative to the floor does the platform recoil?
(c) (ii), (iv) (d) (iii), (iv) (a) (mv0)/(M – m) (b) (mv0)/(M + m)
48. A shell is fired from a cannon with a velocity 20 m/s at (c) (Mv0)/(M – m) (d) None
an angle 60° with the horizontal. At the highest point it 54. A man weighing 80 kg is standing in a trolley weighing
explodes into three pieces of equal masses. One of the 320 kg. The trolley is resting on frictionless horizontal
pieces retraces its path to the cannon, the second piece rails. If the man starts walking on the trolley with a
moves vertically up with speed 40 m/s, the third piece speed of 1 m/s, then after 4 sec his displacement (in m)
will move with velocity (immediately after the explosion) relative to the ground will be
(a) 20 m/s in a horizontal direction
(b) 40 m/s at angle 45° with horizontal in an upward 1 m/s
direction
(c)
40 2 m/s at angle 45° with horizontal in an up- v
ward direction
(d)
40 2 m/s at angle 45° with horizontal in the down-
ward direction
49. A ball is released from height 80 m. When the ball is at
height 60 m, it explodes in two parts of mass ratio 1 : 2.
6 Physics

55. A body of mass M at rest explodes into three pieces, with a speed of v0. The energy released in explosion
two of which of mass M/4 each are thrown off in x
perpendicular directions with velocities of 3 m/s and 4 mv0 2 . Find (x +y).
y
m/s respectively. The third piece will be thrown off with
a velocity (in m/s) of
Third Law of Motion (Using Free Body Diagrams)
56. A shell of mass 20 kg at rest explodes into two fragments
whose masses are in the ratio 2 : 3. The smaller fragment 65. When a car and a truck runs into each other, which
moves with a velocity of 6 m/s. The kinetic energy (in J) exerts more force?
of the larger fragment is (a) Car
57. A bomb of mass 3.0 kg explodes in air into two pieces (b) Truck
of masses 2.0 kg and 1.0 kg. The smaller mass goes at a (c) Both exerts equal force
speed of 80 m/s. The total energy (in kJ) imparted to the (d) No force exists between them
two fragments is 66. How many force pairs act on a box kept on the head of
58. A bomb of mass 9 kg explodes into 2 pieces of mass 3 a person?
kg and 6 kg. The velocity of mass 3 kg is 1.6 m/s, the (a) 1 (b) 2
K.E. (in J) of mass 6 kg is (c) 3 (d) 4
59. A shell of mass 200 g is ejected from a gun of mass 4 67. We can derive Newton’s
kg by an explosion that generates 1.05 kJ of energy. The (a) second and third laws from the first law
initial velocity (in m/s) of the shell is (b) first and second laws from the third law
60. A bullet of mass 50 g is fired with a speed of 200 m/s (c) third and first laws from the second law
from a gun of mass 2 kg. Find the recoil velocity (in (d) All the three laws are independent of each others
m/s) of the gun.
68. When we jump out a boat standing in water it moves
61. A bomb of mass 4 m explodes into two parts of mass
(a) forward (b) backward
ratio 1:3. If the K.E. of smaller fragment is K, find the
(c) sideways (d) none of these
K
K.E. of the larger fragment is . Find x. 69. A man is at rest in the middle of a pond on perfectly
x
62. As shown, a small body of mass 50 g placed over a smooth ice. He can get himself to the shore by making
larger mass 1950 g whose surface is horizontal near the use of Newton’s
smaller mass and gradually curves to become vertical. (a) first law (b) second law
The smaller mass is pushed on the longer one at a speed (c) third law (d) all the laws
20 m/s and the system is left to itself. All surfaces are 70. Newton’s third law is equivalent to the
smooth. Find the speed (in m/s) of the larger block when
(a) law of conservation of linear momentum
the smaller block is moving on the vertical part.
(b) law of conservation of angular momentum
(c) law of conservation of energy
(d) law of conservation of energy and mass
71. A machine gun is mounted on a 2000 kg car on a
horizontal frictionless surface. At some instant the gun
v fires bullets of mass 10 gm with a velocity of 500 m/sec
m with respect to the car. The number of bullets fired per
second is ten. The average thrust (in N) on the system is
(a) 30 (b) 40
63. A U238nucleus, initially at rest, emits an alpha particle (c) 50 (d) 60
with a speed v0. The recoil speed of the residual nucleus 72. In the situation as shown in
x figure if acceleration of B is
Th234 is v . Find ( y-x). Assume that the mass of a
y 0 a then find the acceleration
nucleus is proportional to the mass number. of A (B always remains
64. A bomb of mass 5m initially at rest explodes and breaks horizontal)
into three pieces of masses in the ratio 1 : 1 : 3. The two (a) a sin q (b) a cot q
pieces of equal mass fly off perpendicular to each other (c) 2a tan q (d) 2a cos q
Newton’s Laws of Motion 7

73. A body A of mass m1 exerts a force on another body 77. Two smooth spheres each of radius 5 cm and weight W
B of mass m2. If the acceleration of B be a2, then the rest one on the other inside a fixed smooth cylinder of
acceleration (in magnitude) of A is radius 8 cm. The reactions between the spheres and the
m vertical side of the cylinder are:
(a) 2 a2 (b) m1m2a2
m1
m
(c) 1 a2 (d) (m1 + m2)a2
m2

74. N bullets each of mass m kg are fired with a velocity v


ms–1 at the rate of n bullets per second upon a wall. The
reaction offered by the wall to the bullets is given by
(a) W/4 and 3W/4 (b) W/4 and W/4
Nmv (c) 3W/4 and 3W/4 (d) W and W
(a) nmv (b)
n 78. A body of mass 1 kg lies on smooth inclined plane. The
Nm Nv block of mass m is given force F = 10 N horizontally
(c)
n (d)
n
v m as shown. The magnitude of net normal reaction on the
block is:
75. A helicopter is moving to the right ata constant horizontal

velocity. It experiences three forces Fgravitational , Fdrag and m = 1 kg

force on it caused by rotor Frotor . Which of the following F = 10 N
diagrams can be correct free body diagram representing
45°
forces on the helicopter?

10
(a) 10 2 N (b) N
2
(c) 10 N (d) none of these
79. In the figure shown P is a plate on which a wedge B is
placed and on B a block A of mass m is placed. The plate
(a) (b) is suddenly removed and system of B and A is allowed
to fall under gravity. Neglecting any force due to air on
A and B, the normal force on A due to B is
A
m

(c) (d) q B
P

mg
(a) (b) mg cos q
cos q
76. As shown to the right, two blocks with masses m and 2 mg
(c) zero (d)
M (M > m) are pushed by a force F in both Case I and cos q
Case II. The surface on which blocks lie, is horizontal 80. A bullet of mass m moving with velocity v0 hits
and frictionless. Let RI be the force that m exerts on M a wooden plank A of mass M placed on a smooth
in case I and RII be the force that m exerts on M in case horizontal surface. The length of the plank is . The
II. Which of the following statements is true? bullet experiences a constant resistive force F inside the
block. The minimum value of v0 such that it is able to
come out of the plank is

F /m 2 F  ( M + m)
(a) (b)
RI = RII and is not equal to zero or F
(a) M 2 Mm
RI = RII = F
(b)
RI < RII
(c) 2F m F ( M + m )
(c) 2 (d)
RI > RII
(d) M Mm
8 Physics

81. A machine gun fires a bullet of mass 40 g with a velocity


12 ms–1. The man holding it can exert a maximum force
of 144 N on the gun. How many bullets can he fire per
g
second at the most? m
a0 = 4

82. System is shown in the figure. Velocity of sphere A is 2m


9 m/s. Find the speed (in m/s) of sphere B.
87. The elevator shown in the figure is descending with
an acceleration of 2 m/s2. The mass of the block A is
0.5 kg. The force (in N) exerted by the block A on the
block B is

A 2 m/s2
83. A body of mass 2.0 kg is placed on a smooth horizontal
B
surface. Two forces F1 = 20 N and F2 = N are acting
on the body in directions making angles of 30° and 60° 88. If contact force between 2 kg and 4 kg is f1 newton
to the surface. The reaction of the surface (in N) on the and between 4 kg and 6 kg is f2 newton, then find out
body will be (f1 + f2).
F2
F1
24 N 6 kg 12 N
30° 60° 2 kg 4 kg
2 kg

84. Two blocks of masses m and 2m are placed on a smooth 89. A cylinder of mass 10 kg is resting between two
horizontal surface as shown in the figure. Find the frictionless inclined surfaces AB and AC, and is
acceleration (in m/s2) of each block and normal reaction attached to a vertical string PQ whose other end Q is
between two blocks if a horizontal force F0 = 3N is fixed to the ceiling as shown in the figure. If forces,
applied on m. cylinder applies to surface AC and AB are 30 N and 40
N, respectively, then the tension (in N) in the string is
(g = 10 m/s2)
m 2m
Q
C B
P
85. Three blocks of masses 2 kg, 3 kg and 5 kg are placed
in contact as shown in the diagram. A horizontal
force of 30 N is applied on 2 kg block. The contact 37°
53°
force between (i) 2 kg and 3 kg block and (ii) 3 kg and
A
5 kg block is x newton and y newton, respectively.
x 90. A perfect smooth sphere A of mass 2 kg is in contact
Find y . with a rectangular block B of mass 4 kg and vertical
wall as shown in the figure. All the surfaces are
smooth. Find the normal reaction (in N) by the
30 N 3 kg 5 kg vertical wall on sphere A.
2 kg

86. Consider the arrangement as shown in the figure. The


lift is moving with acceleration g/4 in vertically B
downward direction. The ratio of contact force between
the blocks between the lower block and the floor of lift A
1
is . Find x. 37°
x
Newton’s Laws of Motion 9

91. In the figure shown, the normal reaction between 2 kg


and 1 kg is a newton and normal reaction between 3 kg
and 2 kg is b newton. Find (a + b). ( g = 10 m/s2)
m
8 N
=1
F2
g M
g 1 k oth
2k o
g Sm
0 N 3k
=6
F1 θ = 30°
(a) ( M + m) g (b) ( M - m) g
92. In the figure shown, a rod of length l(< 2R) and mass m
2 2
is kept horizontally inside a smooth spherical shell as
(c) 2(M + m)g (d) None
shown in the figure. The normal reaction when l = R is
mg 96. In the previous problem if M = 80 kg and m = 40 kg and
. Find y. (g = Acceleration due to gravity) the boy pulls the rope such that the system (boy + box)
y moves upward with acceleration 5 m/s2, find the ratio of
force exerted by the boy and contact force between the
R
boy and the box.
(a) 3 : 1 (b) 4 : 1
l (c) 5 : 1 (d) 6 : 1
97. A circular rope of weight W and radius r = R/2 is resting
What’s with all the Tension in the Room? on a smooth sphere of radius R. The tension in the rope is
93. A lift of total mass M is raised by cables from rest
r Rope
through a height h. The greatest tension which the q R
cables can safely bear is nMg. The maximum speed of
lift during its journey if the ascent is to made in shortest R
time is
Ê n + 1ˆ
(a) 2 gh Á (b) 2gh (a)
W (p 6 ) (b)
W /(p 12 )
Ë n ˜¯
(c)
W (p 10 ) (d) none of these
Ê n ˆ Ê n - 1ˆ
(c) 2 gh Á (d) 2 gh Á
Ë n + 1˜¯ Ë n ˜¯ 98. Two blocks of masses m1 and m2, which are connected
with a light string, are placed over a frictionless pulley.
94. A uniform sphere of weight w and radius 3 m is being held This set up is placed over a weighing machine, as shown.
by a string of length 2 m. attached to a frictionless wall as Three combination of masses m1 and m2 are used, in first
shown in the figure. The tension in the string will be: case m1 = 6 kg and m2 = 2 kg, in second case m1 = 5 kg
and m2 = 3 kg and in third case m1 = 4 kg and m2 = 4 kg.
Masses are held stationary initially and then released. If the
Wall readings of the weighing machine after the release in three
cases are W1, W2 and W3 respectively then:

(a) 5w/4 (b) 15w/4


(c) 15w/16 (d) none of these
95. Consider the situation as shown in the figure. The boy
of mass M holds the light rope and the system is at rest.
If m is mass of the box, contact force between the boy
and the box.
10 Physics

W1 > W2 > W3 (b)


(a) W1 < W2 < W3
W1 = W2 = W3 (d)
(c) W1 = W2 < W3
99. Figure shows a man of mass 50 kg standing on a light
weighing machine kept in a box of mass 30 kg. The box
is hanging from a pulley fixed to the ceiling through a
light rope, the other end of which is held by the man
himself. If the man manages to keep the box at rest, the
weight shown by the machine is (a) 0 (b) •
4F 2F
(c) (d)
m m
102. In the above question, find out the magnitude of net
force exerted by the pulley on the rod R1:
(a) 2 F (b) F
F
(c) 2 2 F (d)
2
103. In the arrangement shown in the figure a rod and a block
are connected by a light string. At certain instant the
block C is moving down with the velocity 3 m/s.
What is the velocity of end A (vA)?
(a) 10 N (b) 100 N
(c) 800 N (d) 200 N Wall
A
vA
100. Two blocks of masses m 1 and m 2 are connected as
shown in the figure. The acceleration of the block
m 2 is: B
30°
Floor
C
v = 3 m/s

3
(a) m/s (b) 3 m/s
2
(c) 3 m/s (d) None of these
104. Three blocks 1, 2 and 3 are arranged as shown in the
figure. The velocities of the blocks v1, v2 and v3 are
shown in the figure. What is the relationship between
v1, v2 and v3?
m g m1 g
(a) 2 (b) v1 v2
m1 + m2 m1 + m2
1 2

4m g - m1 g m2 g
(c) 2 (d)
m1 + m2 m1 + 4m2

101. In the given arrangement, mass of the block is M and


the surface on which the block is placed is smooth. 3
v3
Assuming all pulleys to be massless and frictionless,
strings to be inelastic and light, R1, R2 and R3 to be light 2v1 + v2 = v3 (b)
(a) v1 + v2 = v3
supporting rods, then acceleration of point ‘P’ will be
(A is fixed): v1 + 2 v2 = v3
(c) (d) None of these
Newton’s Laws of Motion 11

105. Find the velocity of ring B (VB) at the instant shown.


The string is taut and inextensible:

vB 30°
B
vA = 1 m/s a 3 m3
60°

a2 a1
1 3 m2
(a) m/s (b) m/s m1
2 4
1 2 a1 + 2 a2 + a3 = 0 (b) 2 a1 - 2 a2 + a3 = 0
(a)
(c) m/s (d) 1 m/s
4
106. The ratio of acceleration of pulley to the acceleration of 2 a1 - 2 a2 - a3 = 0 (d) 2 a2 - 2 a1 + a3 = 0
(c)
the block is (string is inextensible) 110. In the figure acceleration of A is 1 m/s2 upwards,
acceleration of B is 7 m/s2 upwards and acceleration of
F
m C is 2 m/s2 upwards. The acceleration of D will be

(a) 0.5 (b) 2


(c) 1 (d) None of these
1 07. In the arrangement shown, the pulleys and the strings
are ideal. The acceleration of block B is
Q

P
C
m A
D
B
m B
A

(a) g/5 (b) g/2 (a) 7 m/s2 downwards (b) 2 m/s2 downwards
(c) 2g/5 (d) 2g/3 (c) 10 m/s2 downwards (d) 8 m/s2 downwards
108. In each of the three arrangements, the block of mass m1
111. Consider the situation shown in figure. All the surfaces
is being pulled left with constant velocity. There is no
are smooth. The tension in the string connected to 2 m is
friction anywhere. The strings are light and inextensible
and pulleys are massless. The ratio of the speed of the
2m
block of mass m2 in the three cases respectively is:

m1 m1 m1 m

mg 4 mg
m2 (a) (b)
m2 3 3
m2
2 mg
mg
(c) (d)
(a) (b) (c) 3
(a) 2 : 1 : 4 (b) 2 : 4 : 1 112. In the figure shown, lower pulley is free to move in
(c) 4 : 2 : 1 (d) Cannot be calculated vertical direction only. Block A is given a uniform
1 09. If the blocks are moving as shown in the figure the velocity u as shown, what is velocity of block B as a
relation between a1, a2 and a3 will be function of angle q.
12 Physics

1 2
(a) m/s 2 (b) m/s 2
3 3
4 5
(c) m/s 2 (d) m/s 2
3 3
116. A balloon is tied to a block. The mass of the block is 2
u kg. The tension of the string between the balloon and the
u cos q (b)
(a) block is 30 N. Due to the wind, the string has an angle q
cos q
relative to the vertical direction, cos q = 4/5 and sin q =
u [1 + sin q ] u [1 + cos q ] 3/5. Assume the acceleration of gravity is g = 10 m/s2.
(c) (d)
cos q sin q Also assume the block is small so the force on the block
113. A block of mass m and wedge M is arranged as shown from the wind can be ignored. Then the x-component
in the figure. and the y-component of the acceleration a of the block.

37°

Initially the system is kept stationary. At certain time


system is released from rest. If acceleration of M is
found to be A = 5 m/s2 towards right, then find net
(a) 9 m/s2, 2 m/s2 (b) 9 m/s2, 12 m/s2
acceleration of m with respect to ground.
(c) 18 m/s2, 2 m/s2 (d) 18 m/s2, 12 m/s2
(a) 2 5 m/s2 (b) 3 m/s2
117. In the system shown in figure mB = 4 kg, and
3 5 m/s2
(c) (d) none of these mA = 2 kg. The pulleys are massless and friction is
114. A perfectly straight portion of a uniform rope has mass absent everywhere. The acceleration of block A is
M and length L. At end A of the segment, the tension in
the rope is TA and at end B it is TB (TB > TA). Neglect
effect of gravity and no contact force acts on the rope
in between points A and B. The tension in the rope at a B
distance L/5 from end A is A
30°
(a) TB – TA (b) (TA + TB)/5
(c) (4TA + TB)/5 (d) (TA – TB)/5 (a) 10/3 m/s2 (b) 20/3 m/s2
2
(c) 2 m/s (d) 4 m/s2
115. A rope of negligible mass passes over a pulley of
negligible mass attached to the ceiling, as shown in 1 18. Two massless rings slide on a smooth circular loop of
figure. One end of the rope is held by Student A of the wire whose axis lies in a horizontal plane. A smooth
mass 70 kg, who is at rest on the floor. The opposite massless inextensible string passes through the rings,
end of the rope is held by Student B of mass 60 kg, which carries masses m1 and m2 at the two ends and
who is suspended at rest above the floor. The minimum mass m3 between the rings. If there is equilibrium when
acceleration a0 with which the Student B should climb the line connecting each ring with centre subtends an
up the rope to lift the Student A upward off the floor. angle 30° with vertical as shown in figure. Then the
ratio of masses are
Newton’s Laws of Motion 13

m1 = 2 m2 = m3 (b)
(a) 2 m1 = m2 = 2 m3 (c) 8 m/sec and 12 m/sec
(d) 12 m/sec and 20 m/sec
m1 = m2 = m3
(c) (d) None of these 122. For the pulley system shown, each of the cables at A and
119. In the arrangement shown in the figure pulleys A and B is given a velocity of 2 m/s in the direction of the arrow.
B are massless and the thread is inextensible. Mass of Determine the upward velocity v (in m/s) of the load m.
pulley C is equal to m. If friction in all the pulleys is B A
negligible, then:

(a) tension in thread is equal to (1/2) mg 123. The system starts from rest and A attains a velocity of
(b) acceleration of pulley C is equal to g/2 (downward) 5 m/s after it has moved 5 m towards right. Assuming
(c) acceleration of pulley A is equal to g (upward) the arrangement to be frictionless everywhere and
(d) acceleration of pulley A is equal to 2g (upward) pulley and strings to be light, the value of the constant
1 20. In the figure shown C is a fixed wedge on horizontal force F (in N) applied on A is:
surface. Blocks A and B are of masses m and 2 m
respectively are kept as shown in figure. They can slide A 6 kg F
along the inclined plane smoothly. The pulley and string
are massless. Take q = 30° and g = 10 m/s2. The inclined
B 2 kg
planes are very long. A and B are released from rest. 2
seconds after the release, B is caught for a moment and 124. In the figure shown blocks A and B are kept on a wedge
released again. Find out the speed of A just before the C. A, B and C each have mass m. All surfaces are
instant when the string becomes tight again. smooth. Find the acceleration (in m/s2) of C.

9
(a) Zero (b) m/s
5 125. As shown in the figure, two blocks of masses 4 kg and
5 kg are at rest on the ground. The masses are connected
12 12
(c) m/s (d) m/s by a massless string passing over a smooth and light
5 7 pulley. A 200 N force is applied on the pulley. Find the
121. A system is shown in figure. All contact surfaces are difference in accelerations (in m/s2) of the blocks.
smooth and string is tight and inextensible. Wedge A
moves towards right with speed 10 m/s and velocity of 200 N
B relative to A is in downward direction along the incline
having magnitude 5 m/s. The horizontal and vertical
component of velocity of Block C respectively are

A 4 kg 5 kg B

(a) 12 m/sec and 18 m/sec 126. If the given system is in equilibrium, then find the ratio
(b) 14 m/sec and 26 m/sec of m1 to m2.
14 Physics

131. In the following arrangement, the system is initially


at rest. The 5 kg block is now released. Assuming the
pulleys and strings to be massless and smooth, find the
ratio of the acceleration of block A to that of C.

m1 m2 5 kg A
10 kg B C 8 kg
127. A flexible chain of weight 10 N hangs between two
fixed points A and B which are at the same horizontal
level. The inclination of the chain with the horizontal at 132. In the given figure, what is the minimum possible
both the points of support is 45°. Find the tension (in N) tension (in N) in string 2?
of the chain at the mid point.
128. In the figure shown, both the blocks are released from 1 2
rest. The time (in s) to cross each other is
37° θ

1 kg

T T 133. In the system shown, block A is of 4.0 kg mass


and blocks B and C are of 3 kg mass each. Find the
a
2m acceleration (in m/s2) of block C, if the system is set free.
(g = 10 m/s2)
4m
4g

1g

129. In the arrangement shown in figure, m1 = 1 kg, m2 = 2


kg. Pulleys are massless and strings are light. For what
value of M (in kg) the mass m1 moves with constant
velocity? (Neglect friction) A
B C
M
1
134. A horizontal force F = 2 N is applied on the system
shown in the figure. All surfaces are smooth. All pulleys
and strings are ideal. Mass of blocks A and B each is 1
2 kg. What is the acceleration (in m/s2) of B with respect
to A?
m2
m1 F

130. In the arrangement shown in the figure, all the surfaces


are frictionless. The masses of the blocks are m1 =
20 kg and m2 = 30 kg. If F = 180 N, then the ratio of
acceleration of mass m2 to that of m1 will be A

m1 B

m2 F
135. A 50 kg person stand on a 25 kg platform. He pulls
on the rope which is attached to the platform via the
Newton’s Laws of Motion 15

frictionless pulleys as shown in the figure. The platform 139. In the figure shown, if block A has a velocity of 60 cm/s
moves upwards at a steady rate if the force (in N) with to the right, then determine the velocity (in cm/s) of
which the person pulls the rope is ­cylinder B.

136. One end of a string is connected to top of the fixed vertical 140. In the system shown, the floor is frictionless. Find the
ring as shown in the figure. The other end is connected acceleration (in m/s2) of point P.
to a bead of mass 3 kg free to slide smoothly on the P
5N
ring. If the string is taut in the shown configuration, then
tension (in newton) in it is equal to (Take q = 37°) 5 kg
5 kg

Springs: Playing with Slinky


141. A block of mass 10 kg is suspended through two light
spring balances as shown in figure.
θ

Bead

137. Two monkeys (1) and (2) of same mass m = 1 kg are


hanging on the strings such that the block of mass 2 kg
remains at rest. It is given that monkey (2) is just holding
the string. Find the acceleration (in m/s2) of monkey (1)
with respect to his rope. (Take g = 10 m/s2) (a) Both the scales will read 10 kg
(b) Both the scales will read 5 kg
(c) The upper scale will read 10 kg and the lower zero
(d) The readings may be anything but their sum will be
10 kg
(1)
142. A spring balance and a physical balance are kept in a lift.
m
(2) In these balances equal masses are placed. If now the lift
2 kg
starts moving upwards with constant acceleration, then
m
(a) The reading of spring balance will increase and the
equilibrium position of the physical balance will
138. The given figure shows that mass m has a velocity of disturb
0.5 m/s upwards and the velocity of mass M is 1 m/s (b) The reading of spring balance will remain un-
downwards. If vP is the velocity (in m/s) of pulley as changed and physical balance will remain in equi-
seen from ground, then find 8|vP|. librium
vp
(c) The reading of spring balance will decrease and
physical balance will remain in equilibrium
(d) The reading of spring balance will increase and the
physical balance will remain in equilibrium
0.5 m/s
m 1 43. An ideal spring is compressed and placed
M horizontally between a vertical fixed wall and a
1 m/s block free to slide over a smooth horizontal table
16 Physics

top as shown in the figure. The system is released


from rest. The graph which represents the relation
between the magnitude of acceleration ‘a’ of the
block and the distance ‘x’ travelled by it (as long as
the spring is compressed) is

x1 = 0, x3 > x2 (b)
(a) x2 > x1 > x3
x3 > x1 > x2 (d)
(c) x1 > x2 > x3
(a) (b) 146. Mass m shown in the figure is in equilibrium. If it is
displaced further by x and released find its acceleration
just after it is released. Take pulleys to be light and
smooth and strings light.
(c) (d)

144. A block ‘A’ of mass ‘m’ is attached at one end of a light


spring and the other end of the spring is connected to
another block ‘B’ of mass 2 m through a light string
as shown in the figure. ‘A’ is held and B is in static 4 kx 2 kx
equilibrium. Now A is released. The acceleration of A (a) (b)
5m 5m
just after that instant is ‘a’. In the next case, B is held
and A is in static equilibrium. Now when B is released, 4kx
(c) (d) none of these
its acceleration immediately after the release is ‘b’. m
The value of a/b is: (Pulley, string and the spring are 147. Initially the spring is undeformed. Now the force
massless) F is applied to B as shown in the figure. When the
displacement of B w.r.t. A is x towards right in some
time then the relative acceleration of B w.r.t. A at that
moment is:
A B
k F
m m

Smooth

F F - kx
(a) (b)
2m m
F - 2 kx
(c) (d) none of these
m
148. Two blocks A and B of masses 2 m and m, respectively,
(a) 0 (b) undefined are connected by a massless and inextensible string. The
whole system is suspended by a massless spring as shown
1
(c) 2 (d) in the figure. The magnitudes of acceleration of A and B
2 immediately after the string is cut, are respectively:
145. Same spring is attached with 2 kg, 3 kg and 1 kg blocks
in three different cases as shown in figure. If x1, x2
and x3 be the extensions in the spring in these cases
then (Assume all the blocks to move with uniform
acceleration)
Newton’s Laws of Motion 17

g g 152. The readings shown in two spring balances S1 and S2 are


(a)
g, (b) ,g 90 kg and 30 kg, respectively, and the lift is accelerating
2 2
upwards with an acceleration of 10 m/s2. If the mass is
g g stationary with respect to the lift, then the mass (in kg)
g, g (d)
(c) ,
2 2 of the block will be
149. The block shown in the figure is in equilibrium. Find 2
10 m/s
acceleration of the block just after the string is cut.

S1

S2
(a) 3g/5 (b) 4g/5
(c) 4g/3 (d) None
150. Two blocks A and B of masses m and 2m respectively are 153. The tension (in N) in the spring is
held at rest such that the spring is in natural length. Find 5N 5N
out the accelerations of blocks A and B respectively just
after release (pulley, string and spring are massless). 154. The masses of 10 kg and 20 kg respectively are
connected by a massless spring as shown in figure. A
force of 200 N acts on the 20 kg mass. At the instant
shown, the 10 kg mass has acceleration 12 m/sec2. What
is the acceleration (in m/s2) of 20 kg mass?

g g
g Ø, g Ø (b)
(a) Ø, ≠
3 3
155. Two blocks of mass 2 kg are connected by a
(c) 0, 0 (d) g Ø, 0 massless ideal spring of spring constant K = 10
N/m. The upper block is suspended from roof
151. Figure shows a 5 kg ladder hanging from a string by a light string A. The system shown is in
that is connected with a ceiling and is having a spring equilibrium. The string A is now cut, the
balance connected in between. A boy of mass 25 kg is acceleration (in m/s2) of upper block just after
climbing up the ladder at acceleration 1 m/s2. Assuming the string A is cut will be (g = 10 m/s2)
the spring balance and the string to be massless and the
spring to show a constant reading, the reading (in kg) of 156. In the setup shown, a 200 N block is supported in
the spring balance is: (Take g = 10 m/s2) equilibrium with the help of strings and a spring. At
point O, the strings are knotted. If the extension in the
spring is 4 cm, then the force constant (in N/m) of the
spring is closest to (g = 10 m/s2)

37º 53º

200 N
18 Physics

157. From the fixed pulley, masses 2 kg, 1 kg and 3 kg are


suspended as shown in the figure. Find the extension
(in m) in the spring if spring constant k = 100 N/m.
(Neglect oscillations due to spring)

2 kg
1 kg
(a) - (30 iˆ + 40 ˆj )
30 iˆ + 40 ˆj (b)

k (c) -(40 iˆ + 30 ˆj )
40 iˆ + 30 ˆj (d)
3 kg 161. Three forces starts acting
 simultaneously on a particle
moving with velocity v . These forces are represented in
Problem Solving Using Newton’s Laws
magnitude and direction by the three sides of a triangle
ABC (as shown). The particle will now move with velocity
158. An elevator is accelerating upwards with an acceleration
of 6 m/s2. Inside it a person of mass 50 kg is standing
on a weighing machine which is kept on an inclined
plane having angle of inclination 60°. The reading of
the weighing machine is:

a = 6 m/s2

(a) v remains unchanged

(b) Less than v

(c) Greater than v
Weighing machine 
60° (d) v in the direction of the largest force BC
162. Consider the three cases given in figures shown. Assume
(a) 40 kg (b) 160 kg the friction to be absent everywhere and the pulleys to
(c) 80 kg (d) 50 kg be light; the string connecting the blocks to other block
1 59. A body of 2 kg has an initial speed 5 ms–1. A force or fixed vertical wall to be light and inextensible. Let
acts on it for some time in the direction of motion. The TA, TB and TC be the tension in the strings in figure A,
force–time graph is shown in the figure. The final speed figure B and figure C respectively. Then pick the correct
of the body. comparision between the given tensions (for the instant
shown) from options below.
F(N)

2.5

0 2 4 4.5 6.5 t(s)

(a) 9.25 ms–1 (b) 5 ms–1


(c) 14.25 ms–1 (d) 4.25 ms–1
160. A particle of mass m = 5 kg, is momentarily  at rest at
x = 0 at t = 0.It is acted upon by
 two forces F 1 and F2 . It is
given that F1 = 70 N ˆj and F2 is unknown. The particle

experiences a constant acceleration  a , in the direction
as shown. What third force, F3 , is required to make
the acceleration of the particle zero? (Note: sin q = 4/5, TA = TB = TC (b)
(a) TB =TC < TA
cos q = 3/5, and tan q = 4/3. Neglect gravity.) TA < TB < TC (d)
(c) TB < TC < TA
Newton’s Laws of Motion 19

163. Consider the system as shown in the figure. The (a) F1 = 3 N, F2 = 5 N, F3 = 9 N


pulley and the string are light and all the surfaces are (b) F1 = 3 N, F2 = 5 N, F3 = 1 N
frictionless. The tension in the string is (g = 10 m/s2). (c) F1 = 3 N, F2 = 5 N, F3 = 15 N
(d) F1 = 3 N, F2 = 5 N, F3 = 6 N
167. Two weights W1 and W2 in equilibrium and at rest are
W
suspended as shown in figure. Then the ratio 1 is:
W2

(a) 0 N (b) 1 N
(c) 2 N (d) 5 N
164. In the shown mass pulley system, pulleys and string are
massless. The one end of the string is pulled by the force
F = 2mg. The acceleration of the block will be
(a) 5/4 (b) 4/5
(c) 8/5 (d) none of these
168. Four identical metal butterflies are hanging from
a light string of length 5l at equally placed points
F = 2 mg as shown. The ends of the string are attached to a
horizontal fixed support. The middle section of the
string is horizontal. The relation between the angle q1
m and q2 is given by
(a) g/2 (b) 0
(c) g (d) 3g
165. A weight W is supported by two strings inclined at
60° and 30° to the vertical. The tensions in the strings
are T1 and T2 as shown. If these tensions are to be
determined in terms of W using a triangle of forces, (a) sin q1 = 2 sin q2
which of these triangles should you draw? (block is in (b) 2 cos q1 = sin q2
equilibrium)
(c) tan q1 = 2 tan q2
T2 T1 (d) q2 < q1 and no other conclusion can be derived.

60°
169. A 50 kg person stands on a 25 kg platform. He pulls
30°
on the rope which is attached to the platform via the
frictionless pulleys as shown in the figure. The platform
moves upward at a steady rate if the force with which
W
the person pulls the rope is:

(a) (b)

(c) (d)
(a) 500 N (b) 250 N
(c) 25 N (d) 50 N
166. Which of the following sets of concurrent forces may be 170. Find the tension in the string AB loaded with weight W
in equilibrium? at the middle, when AB is horizontal:
20 Physics

(a) 30° (b) 60°


(c) 90° (d) 120°
175. The adjacent figure is the part of a horizontally stretched
net. Section AB is stretched with a force of 10 N. The
tensions in the sections BC and BF are
(a) zero (b) W
E
(c) W/2 (d) infinity
150° 150°
171. In the figure, at the free end of the light string, a force
D
F is applied to keep the suspended mass of 18 kg at
rest. Then the force exerted by the ceiling on the system
(assume that the string segments are vertical and the 120°
C
pulleys are light and smooth) is: (g = 10 m/s2) G
B
F H

120°

(a) 10 N, 11 N
(b) 10 N, 6 N
(c) 10 N, 10 N
(d) Cannot calculate due to insufficient data
176. In the arrangement shown in figure the ends P and Q of
an unstretchable string move downwards with uniform
speed U. Pulleys A and B are fixed. Mass M moves
(a) 60 N (b) 120 N upwards with a speed
(c) 180 N (d) 240 N
A B
172. A block of mass 40 gm is moving with a constant q q
velocity of 2 cm/s on a horizontal frictionless table. The
force on the table by the block is (Take g = 10 ms–2) M
(a) 0.40 N downwards P Q

(b) 0.40 N upwards


(c) net force on the table by block is zero U cos q
(a) 2U cos q (b)
(d) none of these
2U U
173. A block of mass m is placed on a smooth inclined plane (c) (d)
cos q cos q
of inclination q with the horizontal. The force exerted
by the plane on the block has a magnitude 177. The pulleys and strings shown in the figure are smooth
(a) mg tan q (b) mg cos q and of negligible mass. For the system to remain in
mg equilibrium, the angle should be
(c) (d) mg
cos q A B
q
174. In the following figure, the pulley P1 is fixed and the
pulley P2 is movable. If W1 = W2 = 100 N, what is the
angle AP2P1? The pulleys are frictionless:
m m
2m

(a) 0° (b) 30°


A P1 (c) 45° (d) 60°
P2
178. A string of negligible mass going over a clamped
W1 pulley of mass m supports a block of mass M as shown
in the figure. The force on the pulley by the clamp is
W2 given by
Newton’s Laws of Motion 21

F = 2 mg
M m 2m m
Case-1 Case-2

aa = ab (b)
(a) aa = ab/2
(a) 2 Mg (b)
2 mg aa = ab/3
(c) (d) aa = 2ab
182. Two blocks are connected by a string as shown in the
(c) ( M + m)2 + m 2 g (d) ( M + m)2 + M 2 g diagram. The upper block is hung by another string. A force
F applied on the upper string produces an acceleration of 2
179. A lift is going up. The total mass of the lift and the
m/s2 in the upward direction in both the blocks. If T and T ¢
passenger is 1500 kg. The variation in the speed of
be the tensions in the two parts of the string, then
the lift is as given in the graph. The tension in the rope
pulling the lift at t = 11th sec will be F
T
Speed in m/s

3.6 2 kg

4 kg
2 10 12

(a) 17400 N (b) 14700 N T = 70.8 N and T ¢ = 47.2 N


(a)
(c) 12000 N (d) Zero
T = 58.8 N and T ¢ = 47.2 N
(b)
180. A boy and a block, both of same mass, are suspended
(c)
T = 70.8 N and T ¢ = 58.8 N
at the same horizontal level, from each end of a light
string that moves over a frictionless pulley as shown. T = 70.8 N and T ¢ = 0
(d)
The boy starts moving upwards with an acceleration 2.5
m/s2 relative to the rope. If the block is to travel a total 183. Refer to the system shown in the figure. The acceleration
distance 10 m before reaching at the pulley, the time of the masses is
taken by the block in doing so is equal to:

T2

10 m 3 kg B
T1
m 1 kg C A 5 kg
m

g g
(a) 8 s (b) 4 s (a) (b)
3 6
10 g g
(c) s (d) 8 s (c) (d)
2 9 12
181. Figure shows two pulley arrangements for lifting a mass 184. In above question, the tension T1 in the string between B
m. In case-1, the mass is lifted by attaching a mass 2 m and C is
while in case-2 the mass is lifted by pulling the other 2g 4g
end with a downward force F = 2 mg, If aa and ab are (a) (b)
9 9
the accelerations of the two masses then (Assume string
6g 10 g
is massless and pulley is ideal) (c) (d)
9 9
22 Physics

185. In above question, the tension T2 in the string between A


and B is
10 g 20 g a
(a) (b) M
9 9
30 g 40 g
(c) (d) (a) g/4 (b) g/2
9 9
(c) 3g/4 (d) 4g
186. M is a fixed wedge. Masses m1 and m2 are connected
by a light string. The wedge is smooth and the pulley is 190. The ratio of tensions in the string connected to the block
smooth and fixed. m1 = 10 kg and m2 = 7.5 kg. When m2 of mass m2 in figure-(a) and figure-(b) respectively is
is just released, the distance it will travel in 2 seconds is (friction is absent everywhere): [m1 = 50 kg, m2 = 80 kg
and F = 1000 N].

m1 m2 m2
m2 m1
30°
F m1 F
(a) 2.8 m (b) 7.5 m
(c) 4.0 m (d) 6.0 m
187. In the figure, the blocks A, B and C of mass m each have (a) 7 : 2 (b) 2 : 7
accelerations a1, a2 and a3 respectively. F1 and F2 are (c) 3 : 4 (d) 4 : 3
external forces of magnitudes 2mg and mg respectively. 1 91. Two small spheres each of mass m connected by a string
Then of length 2l are kept on a smooth horizontal surface. A
vertical force F is applied at the middle of the string.
What is maximum value of F for which the spheres do
not lose contact with the surface?
F

m m m
q q
A B C m
F1 = 2mg 2m
mg
(a) 2mg (b)
F1 = mg
3 mg
(c) (d) 4mg
a1 = a2 = a3
(a) (b) a1 > a3 > a2 2
a1 = a2, a2 > a3 (d)
(c) a1­ > a2, a2 = a3 192. A rod of length  leans by its upper end against a smooth
188. A mass of 5 kg is suspended in equilibrium, by two light vertical wall, while its other end leans against the floor.
inextensible strings S1 and S2 which make angles of 30° The end that leans against the wall moves uniformly
and 45° respectively with the horizontal. Then (Take downward. Then
g = 10 ms–2) y
(a) tension in both the strings is same
(b) tension in S1 is more than that in S2 l
(c) tension in S1 is less than that in S2 uy
ux
(d) sum of tensions in both is equal to 50 N
x
189. With what acceleration a should the box of figure
moving up so that the block of mass M exerts a force (a) the other end also moves uniformly
7 Mg/4 on the floor of the box? (b) the speed of other end goes on decreasing
Newton’s Laws of Motion 23

(c) the speed of outer end goes on increasing 196. A wedge of height ‘h’ is released from rest with a light
(d) the speed of other end first decreases and then in- particle P placed on it as shown. The wedge slides down
creases an incline which makes an angle q with the horizontal.
All the surfaces are smooth, P will reach the surface of
193. A block of mass m lies on wedge of mass M, which
the incline in time:
lies on fixed horizontal surface. The wedge is free to
move on the horizontal surface. A horizontal force of
magnitude F is applied on block as shown, neglecting
friction at all surfaces, the value of force F such that
block has no relative motion w.e.t. wedge will be:
(where g is acceleration due to gravity)

2h 2h
(a) (b)
2
g sin q g sin q cos q

2h 2h
(c) (d) 2
g tan q g cos q
(a) (M + m) g tan q (b) (M + m) g cot q
197. A cylinder rests in a supporting carriage as shown.
m m The side AB of carriage makes an angle 30° with the
(c) ( M + m) g tan q (d) ( M + m) g cot q
M M horizontal and side BC is vertical. The carriage lies on
a fixed horizontal surface and is being pulled towards
194. A particle of mass m is placed on the smooth face of
left with an horizontal acceleration ‘a’. The magnitude
an inclined plane of mass M and slope q which is free
of normal reactions exerted by sides AB and BC of
to slide on a smooth horizontal plane in a direction
carriage on the cylinder be NAB and NBC respectively
perpendicular to the edge. If the particle slides with an
(Neglect friction everywhere). Then as the magnitude
accelerating a, the acceleration of the inclined plane
of acceleration ‘a ‘ of the carriage is increased, pick up
towards right A will be
the correct statement:
m
a
M A
q

ma cos q Mm cos q
(a) (b)
( M + m) ( M + m)

Ma cos q M cos q
(c) (d)
( M + m) ( M + m) (a) NAB increases and NBC decreases.
195. A force F = bt (where b is a constant) is applied at an (b) Both NAB and NBC increase.
angle to a mass m kept on a smooth horizontal plane. (c) NAB remains constant and NBC increases.
The velocity of mass m at the moment of its breaking (d) NAB increases and NBC remains constant.
off the plane is 198. A ball is suspended on a thread from the ceiling of a
car. The brakes are applied and the speed of car changes
from 5 m/sec to 5/3 m/sec during the time interval of 3
seconds. Find the angle that the thread with the ball will
deviate from vertical.
mg 2 cos a g 2 cos a Ê 1ˆ Ê 8ˆ
(a) (b)
2b sin 2 a 2 mb sin 2 a q = tan–1 Á ˜ (b)
(a) q = tan–1 Á ˜
Ë 9¯ Ë 9¯

mg 2 sin 2 a bg 2 cos a Ê 1ˆ Ê 1ˆ
(c) (d) q = cos–1 Á ˜ (d)
(c) q = sin–1 Á ˜
2b cos a 2 m sin a Ë 9¯ Ë 9¯
24 Physics

199. A body of mass m is placed over a smooth inclined is released and the wedge is observed to move with
plane of inclination q, which is placed over a lift which acceleration A on inclined surface as shown. There is no
is moving up with an acceleration a0. Base length of the friction anywhere. The acceleration of block (m) with
inclined plane is L. Calculate the velocity of the block respect to wedge (M) will be
with respect to lift at the bottom, if it is allowed to slide
down from the top of the plane from rest. m
M
(a) 2(a0 + g ) L sin q (b) 2(a0 + g ) L cos q
A q
(c) 2(a0 + g ) L tan q (d) 2(a0 + g ) L cot q
(a) A rightward (b) A cos q rightward
200. A plumb line is suspended from a ceiling of a car (c) A cos q leftward (d) none of these
moving with horizontal acceleration of a. What will be 205. A bob is hanging over a pulley inside a car through a
the angle of inclination with vertical string. The second end of the string is in the hand of
(a) tan–1 (a/g) (b) tan–1 (g/a) a person standing in the car. The car is moving with
–1
(c) cos (a/g) (d) cos–1 (g/a) constant acceleration a directed horizontally as shown
201. A block of mass m is placed on a smooth wedge in figure. Other end of the string is pulled with constant
of inclination q. The whole system is accelerated acceleration a (relative to car) vertically. The tension in
horizontally so that the block does not slip on the the string is equal to
wedge. The force exerted by the wedge on the block a
(g is acceleration due to gravity) will be a
q
(a) mg cos q (b) mg sin q m
(c) mg (d) mg/cos q
202. A block is kept on a frictionless inclined surface
with angle of inclination a. The incline is given an

2 2
m g 2 + a 2 - ma
(a) m g + a (b)
acceleration a to keep the block stationary. Then a is
equal to m g 2 + a 2 + ma (d)
(c) m(g + a)

206. A large cubical shaped block of mass M rests on a fixed


horizontal surface. Two blocks of mass m1 and m2 are
a connected by a light inextensible string passing over
a a light pulley as shown. Neglect friction everywhere.
Then the constant horizontal force of magnitude F that
(a) g (b) g tan a
should be applied to M so that m1 and m2 do not move
(c) g/tan a (d) g cosec a relative to M is:
203. A block of mass 2 kg slides down the face of a smooth
m2
45° wedge of mass 9 kg as shown in figure. The wedge
is placed on a frictionless horizontal surface. Determine
the acceleration of the wedge. F M
m1

2 kg
m2
F=
(a) (m + m2 + M )g
Wedge A m1 1
a
9 kg m1
(b)
F= (m + m2 + M )g
m2 1
11
(a) 2 m/s2 (b) m/s2 m1
2 F=
(c) (m + M )g
m2 1
(c) 1 m/s2 (d) none of these
204. A block of mass m is kept on a wedge of mass M. m2
F=
(d) (m + M )g
Initially the system is held. At certain time the system m1 1
Newton’s Laws of Motion 25

207. A rod AB is shown in figure. End A of the rod is fixed


on the ground. Block is moving with velocity 3 m/s
towards right. The velocity of end B of rod when rod
makes an angle of 60° with the ground is:

g 5g
(a) (b)
3 2
(a) 3 m/s (b) 2 m/s 2g 2g
(c) (d)
(c) 2 3 m/s (d) 3 m/s 3 5

208. In the figure shown, find out the value of q [assume 211. In the figure if block A and wedge B will move with same
string to be tight] acceleration, then the magnitude of normal reaction
between the block and the wedge will be (There is no
1 m/s friction between block and the wedge and the wedge
moves on horizontal surface as shown.)

q 30°
3.25 m/s 3 m/s
A B

3 4
(a) tan–1 (b) tan–1
4 3 (a) 2 mg/cos q (b) 2 mg cos q
(c) mg cos q (d) none of these
(c) tan–1 3 (d) none of these
8 212. A block is placed on a smooth wedge of inclination
angle q with the horizontal. What minimum acceleration
209. A rod can freely rotate in vertical plane about the hinge
should be given to the wedge in the horizontal direction
at its bottom. Two strings tie the top of the rod with
so that the block starts moving up along the incline?
blocks A and B as shown in the figure. At the instant
shown, the rod is vertical and the speed of block A is u
m/s downwards. Find the speed of block B.

(a) >g cot q (b) >g tan q


(c) >g sin q (d) >g cos q
213. A monkey of mass 20 kg is holding a vertical rope. The
rope can break when a mass of 25 kg is suspended from
u
(a) 3 u (b) it. What is the maximum acceleration (in m/s2) with
which the monkey can climb up along the rope?
u 2
(c) (d) u
214. In the given diagram, with what force (in N) must the
2 2 3
man pull the rope to hold the plank in position? Mass of
210. In the figure shown neglecting friction and mass of the man is 80 kg. Neglect the weights of plank, rope and
pulleys, what is the acceleration of mass B? pulley. Take g = 10 ms–2.
26 Physics

219. In the figure shown, blocks A and C start from rest and
move to the right with acceleration aA = 12t m/s2 and
aC = 3 m/s2. Here, t is in seconds. The time (in s) when
block B again comes to rest is

A C

215. A car is moving on a plane inclined at 30º to the


B
horizontal with an acceleration of 9.8 m/s2 parallel to
the plane upward. A bob is suspended by a string from 220. In the figure shown, a block A of mass 2 kg, tied to a
the roof. The angle (in degrees) which the string makes string attached to ceiling, is placed on the wedge B.
with the vertical is: (Assume that the bob does not move The string is parallel to the inclined surface. All the
relative to car) [g = 9.8 m/s2] surfaces are frictionless. If B is given an accelera-
216. A lift is falling with an acceleration 2 m/s2. A ball of tion of 5 m/s2 towards right, the force (in N) on B
mass 100 gm is attached at one end of the string and due to A is
the other end is fixed to the ceiling of the lift. The ball
remains at rest relative to lift. The tension (in N) in the
string is: (g = 10 m/s2)
A 5 m/s
217. A lift is moving upwards with a constant speed of 5 m/s.
The speed of one of the pulleys is 5 m/s as shown. Then B
the speed (in m/s) of second pulley is: 37°

221. A small block of mass m is placed on a plank of mass


2m and length L. A force F is applied as shown in the
figure. The time taken by block A to reach the end of
x mL
plank B is . Find x.
F
A
m
F
B

218. In the figure shown, an inclined plane makes an angle L


30° with the horizontal. A groove OA=5 m cut in the
plane makes an angle 30° with OX. A short smooth 222. In the figure shown, starting from rest, the cable can
­cylinder is free to slide down in the groove due to the be wound onto the drum of the motor at a rate of vA =
influence of gravity. The time (in s) taken by the cylin- (3t2) m/s, where t is seconds. The time needed to lift
der to reach from A to O, if released from rest from A, the load 7 m is 3 k seconds. Find k.
is (g = 10 m/s2)

Cylinder A A D

30° Motor B
30°
O X Load
Newton’s Laws of Motion 27

223. For the situation shown, the acceleration of mass m is F


x
9F
. Find p. y
pm M M

C B
A F
m 227. In the figure shown, pulley and cord have negligible
mass and friction. The system is balanced with equal
masses M on each side as shown (solid line), and then
a small mass m is gently placed to one block at posi-
224. In the figure shown, a spherical surface is fixed on tion A. The combined masses accelerate through a cer-
ground. Its radius is 5 m. A block is placed some tain distance h, and then mass m is gently removed at
height below the top most point. If acceleration of the position B and two equal masses then move on with
block at any instant is 6 m/s2, then find the height (in constant speed v. If the value of v is 10a (in cm/s),
m) of the block above ground at that instant. then find the value of a. (Given: m = 250 g, M = 1 kg,
h = 16.2 cm and g = 10 m/s2)

225. A truck accelerates with 12 m/s2. A bead can slide fric- A m


M
36
tionlessly on a wire of length m fixed on a truck as
5 h
shown in the figure. At an instant the bead is released M
from the lowest point of the wire. Find the time (in sec-
B
ond) taken by it to reach the top of the wire. M

12 m/s2 228. A uniform rope of length L and mass M is placed on a smooth


fixed wedge as shown in the figure. Both ends of rope are at
same horizontal level. If the rope is initially released from
m
Bead /5 rest, then the magnitude of initial acceleration of rope is
36
θ = 37° k (cos a – cos b) Mg .Find k.

226. In the situation given, all the surfaces are frictionless,


pulley is ideal and string is light. If F = Mg/2, then find α β
the magnitude of difference in acceleration (in m/s2) of
both the blocks. (g = 10 m/s2)
28 Physics

ARCHIVE
1. A mass of 10 kg is suspended vertically by a rope from m1 and m2 move with velocities of v3iˆ and v4 iˆ ,
the roof. When a horizontal force is applied on the rope respectively. If m2 = 0.5m1 and v3 = 0.5v1, then v1 is
at some point, the rope deviated at an angle of 45° at v2 v
v4 -
(a) v4 - 2
(b)
the roof point. If the suspended mass is at equilibrium, 4 2
the magnitude of the force applied is (g = 10 ms–2) v4 - v2 (d)
(c) v4 + v2
(a) 200 N (b) 100 N
 (JEE Main 2019)
(c) 140 N (d) 70 N
(JEE Main 2019) 6. A particle of mass ‘m’ is moving with speed ‘2v’ and
collides with a mass ‘2m’ moving with speed v in
2. A particle of mass m is moving in a straight line with
the same direction. After collision, the first mass is
momentum p. Starting at time t = 0, a force F = kt acts
stopped completely while the second one splits into
in the same direction on the moving particle during
two particles each of mass ‘m’, which move at angle
time interval T so that its momentum changes from p to
45° with respect to the original direction. The speed of
3p. Here k is a constant. The value of T is
each of the moving particle will be
p 2p v
(a)
2 (b) (a) (b) 2 2v
k k (2 2)
2k k v
(c) 2 v (d)
(c) (d)
2
p p 2
 (JEE Main 2019)
(JEE Main 2019)
7. A man (mass = 50 kg) and his son (mass = 20 kg) are
3. A spring whose unstretched length is l has a force
standing on a frictionless surface facing each other. The
constant k. The spring is cut into two pieces of
man pushes his son so that he starts moving at a speed
unstretched lengths l1 and l2 where, l1 = nl2 and n is
of 0.70 ms–1 with respect to the man. The speed of the
an integer. The ratio k1/k2 of the corresponding force
man with respect to the surface is
constants, k1 and k2 will be
(a) 0.20 ms -1 (b) 0.14 ms -1
1
n2
(a) 2 (b) 0.47 ms -1 (d)
(c) 0.28 ms -1
n
 (JEE Main 2019)
1
(c) (d) n 8. If 1022 gas molecules each of mass 10–26 kg collide with
n
a surface (perpendicular to it) elastically per second
(JEE Main 2019)
over an area 1 m2 with a speed 104 m/s, the pressure
4. A piece of wood of mass 0.03 kg is dropped from the exerted by the gas molecules will be of the order of
top of a 100 m height building. At the same time, a
(a) 108 N/m2 (b) 104 N/m2
bullet of mass 0.02 kg is fired vertically upward, with 3
(c) 10 N/m 2 (d) 1016 N/m2
a velocity 100 ms–1, from the ground. The bullet gets  (JEE Main 2019)
embedded in the wood. Then the maximum height to
which the combined system reaches above the top of 9. A mass of 10 kg is suspended by a rope of length 4 m,
the building before falling below is (g = 10 ms–2) from the ceiling. A force F is applied horizontally at the
(a) 30 m (b) 10 m mid-point of the rope such that the top half of the rope
(c) 40 m (d) 20 m makes an angle of 45° with the vertical. Then F equals
 (JEE Main 2019) (Take g = 10 ms–2 and the rope to be massless)
(a) 100 N (b) 90 N
5. A body of mass m1 moving with an unknown velocity
(c) 75 N (d) 70 N
of v1iˆ , undergoes a collinear collision with a body of
(JEE Main 2020)
mass m moving with a velocity v2 iˆ . After collision,
2
Newton’s Laws of Motion 29

  ANSWER KEY 
1. (b) 2. (d) 3. (a) 4. (c) 5. (0) 6. (b) 7. (b) 8. (b) 9. (a) 10. (b)
11. (d) 12. (d) 13. (c) 14. (c) 15. (a) 16. (a) 17. (c) 18. (c) 19. (d) 20. (a)
21. (a) 22. (b) 23. (d) 24. (b) 25. (c) 26. (d) 27. (90) 28. (417) 29. (0.12) 30. (10)
31. (0.90) 32. (10) 33. (25) 34. (4) 35. (1) 36. (65) 37. (a) 38. (d) 39. (d) 40. (d)
41. (d) 42. (a) 43. (c) 44. (b) 45. (d) 46. (a) 47. (b) 48. (d) 49. (c) 50. (b)
51. (b) 52. (a) 53. (b) 54. (3.20) 55. (2.50) 56. (216) 57. (4.80) 58. (1.92) 59. (100) 60. (5)
61. (3) 62. (0.50) 63. (115) 64. (7) 65. (c) 66. (a) 67. (c) 68. (b) 69. (c) 70. (a)
71. (c) 72. (d) 73. (a) 74. (a) 75. (c) 76. (c) 77. (c) 78. (a) 79. (c) 80. (b)
81. (3) 82. (12) 83. (25) 84. (2) 85. (1.60) 86. (3) 87. (4) 88. (40) 89. (50) 90. (45)
91. (64) 92. (3) 93. (d) 94. (a) 95. (b) 96. (a) 97. (b) 98. (b) 99. (b) 100. (a)
101. (c) 102. (c) 103. (b) 104. (d) 105. (d) 106. (a) 107. (c) 108. (a) 109. (d) 110. (c)
111. (c) 112. (c) 113. (d) 114. (c) 115. (d) 116. (a) 117. (a) 118. (c) 119. (d) 120. (a)
121. (b) 122. (1.50) 123. (75) 124. (0) 125. (5) 126. (15) 127. (5) 128. (1) 129. (8) 130. (0)
131. (2) 132. (6) 133. (2) 134. (2) 135. (250) 136. (48) 137. (35) 138. (2) 139. (180) 140. (13)
141. (a) 142. (d) 143. (c) 144. (c) 145. (b) 146. (c) 147. (c) 148. (b) 149. (c) 150. (a)
151. (32.50) 152. (30) 153. (5) 154. (4) 155. (20) 156. (3000) 157. (0.20) 158. (a) 159. (c) 160. (b)
161. (a) 162. (d) 163. (d) 164. (d) 165. (c) 166. (d) 167. (a) 168. (c) 169. (b) 170. (d)
171. (d) 172. (a) 173. (b) 174. (d) 175. (c) 176. (d) 177. (c) 178. (d) 179. (c) 180. (b)
181. (c) 182. (a) 183. (c) 184. (d) 185. (d) 186. (a) 187. (b) 188. (c) 189. (c) 190. (c)
191. (a) 192. (b) 193. (c) 194. (a) 195. (a) 196. (a) 197. (c) 198. (a) 199. (c) 200. (a)
201. (d) 202. (b) 203. (c) 204. (b) 205. (c) 206. (b) 207. (b) 208. (a) 209. (d) 210. (d)
211. (a) 212. (b) 213. (2.50) 214. (200) 215. (30) 216. (0.80) 217. (5) 218. (2) 219. (0.50) 220. (10)
221. (8) 222. (56) 223. (3) 224. (9) 225. (2) 226. (5) 227. (6) 228. (0)

Archive
1.
(b) 2. (a) 3. (c) 4. (c) 5. (c) 6. (b) 7. (a) 8. (*) 9. (a)
NEWTON'S LAWS OF MOTION

Displacement of body in 4 sec along OE


First Law of Motion
sx = vxt = 3 ¥ 4 = 12 m
1. (b) Force along OF (perpendicular to OE) = 4 N
Horizontal velocity of apple will remain same but due to retardation
F 4
of train, velocity of train and hence velocity of boy w.r.t. ground \ ay = = = 2 m/s 2
decreases, so apple falls away from the hand of boy in the direction m 2
of motion of the train. Displacement of body in 4 sec along OF
2. (d) 1 2 1
fi s y = u yt + a yt = ¥ 2 ¥ (4) 2 = 16 m [As uy = 0]
Horizontal velocity of ball and person are same so both will cover 2 2
equal horizontal distance in a given interval of time and after
following the parabolic path the ball falls exactly in the hand which \ Net displacement s = s x2 + s 2y = (12) 2 + (16) 2 = 20 m
threw it up.
8. (b)
3. (a)
Net force on the body is zero.
When the string C is stretched slowly, the tension in A is greater
than that of C, because of the weight mg and the former reaches 9. (a)
breaking point earlier. For equilibrium of system, F1 = F22 + F32 , as q = 90°
4. (c) In the absence of force F1,
The force exerted by the locomotive on the wagons is equal to that
Net force
exerted by the wagons on the locomotive. The locomotive moves Acceleration =
forward because of a forward frictional force exerted by the tracks Mass
on its wheels. The backward frictional force exerted by the track on
the wheels of the wagons is much smaller. F22 + F32 F1
= =
m m
5. (0)
Newton’s first law states that an object in motion tends to stay 10. (b)
in motion unless if acted upon by a net force. This means that if     
friction is not being accounted for, there is no net force required to
F1 + F2 + F3 + F4 + F5 = 2(4iˆ ) (i)
keep an object moving if it's in motion. A net force is only required    
to change an object's motion. The 500 kg object is moving at a and F2 + F3 + F4 + F5 = 2(7 ˆj ) (ii)
constant velocity; therefore, there is no net force (0 newtons) acting 
From (i) and (ii), F1 = 8iˆ - 14 ˆj
on the object. 
 F1
Second Law of Motion a1 = = 4iˆ - 7 ˆj
m
6. (b)
Velocity between t = 0 and t = 2 sec fi a1 = 16 + 49 = 65 m/s2

dx 4
fi vi = = = 2 m/s 11. (d)
dt 2 Momentum acquired by the particle is numerically equal to area
Velocity at t = 2 sec, vf = 0 enclosed between the F-t curve and time axis. For the given diagram
Impulse = Change in momentum = m(vf – vi) area in upper half is positive and in lower half is negative (and equal
= 0.1(0 – 2) = –0.2 kg m sec–1 to upper half), so net area is zero. Hence the momentum acquired by
the particle will be zero.
7. (b)
12. (d)
dp
As F=
dt

Ú dp = Ú F dt
Dp = area under F-t graph
2 Solutions

1 18. (c)
Dp = ¥ 8 ¥ 1 - 2 ¥ 8.5 = 3 kg m/s
2 p= 2 mK
Net area is zero. So momentum gained is zero.
1 1 p m 1 1
13. (c) = = =
Impulse = Area between force and time graph and it is maximum for p2 m2 4 2
graph (III) and (IV) 19. (d)
14. (c) p= 2 m1 K1 = 2 m2 K 2

Ú dp = p f - pi = Ú F dt m K 1
1
= 2 =
= Area under the curve. m2 K1 4
pi = 0
Net Area = 16 – 2 – 1 = 13 N-s 20. (a)
For percentage change, use
13
= Vf = = 6.5 iˆ m/s Ê Final value ˆ
2 = Á - 1˜ ¥ 100%
Ë Initial value ¯
[As momentum is positive, particle is moving along positive x-axis.]
15. (a) Ê 300 ˆ
K1 = K, K2 = K Á 1 +
= 4K
F = at; m
dv
= at; Ú mdv = Ú at dt
Ë 100 ˜¯
dt
p2 K2
at 2 ds at 2 = =2
mv = ;m = p1 K1
2 dt 2

at 2
Êp ˆ

2
- 1 ¥ 100 = 100%

Ú mds = Ú 2
dt ÁË p
1
˜¯

at 3 a Ê F ˆ
3 21. (a)
ms = = Á ˜ ; \ S µ F3
6 6Ë a¯ Ê 0.1 ˆ
K1 = K, K2 = K Á 1 +
Ë 100 ˜¯
F µ S1/3
1/2
16. (a) p2 K2 Ê 0.1 ˆ 1 0.1
= = Á1 + =1+ ¥
Change in linear momentum DP = Ú Fdt p1 K1 Ë 100 ˜¯ 2 100

15
Êpˆ Ê p2 ˆ 1
15(v f + u) = Ú 40 cos Á ˜ t dt
Á - 1˜ ¥ 100 = ¥ 0.1 = 0.05%
Ë 10 ¯ Ë p1 ¯ 2
0

15
40 È sin (p / 10) t ˘ 400 22. (b)
vf = -4 + ˙ = - 4 + 15 p (-1) = - 12.5m/s u
15 ÍÎ p / 10 ˚0

17. (c) q

1 2 q
u


u = u cos q iˆ + u sin q ˆj

1

u = u cos q iˆ - u sin q ˆj

2
  
Dp = p - p = - 2 mu sin q ˆj
  2 1
As v = 5t iˆ + 2t ˆj \ a = a x iˆ + a y ˆj = 5iˆ + 2 ˆj 
Dp = Dp = 2 mu sin q = 2 mu sin 30∞

F = ma x iˆ + m( g + a y ) ˆj
1 1
 = 2¥ ¥ 98 ¥ = 49 N-s
\ F = m a x2 + ( g + a y )2 = 26 N 2 2
Newton's Laws of Motion 3

23. (d)  
dv m(v2 - v1 ) 0.4(-20 - 10)
2 F=m = = = 100 N
mv 2 m Ê p ˆ p2 dt dt dt
F= = Á ˜ =
r r Ë m¯ mr by solving dt = 0.12 sec.

24. (b) 30. (10)


During collision of ball with the wall horizontal momentum changes
(vertical momentum remains constant)

u = 2 gh1 , v = 2 gh2
60°
P = mv 30°

Dp = m(v + u)
= m( 2 gh2 + 2 gh1 )
Change in horizontal momentum
\ F=
25. (c) Time of contact
V 2 P cosq 2 mv cosq
2 1
= =
O 0.1 0.1
V 2 ¥ 0.1 ¥ 10 ¥ cos60∞
= = 10 N
From 1 to 2, 0.1
Dp = 2mV
31. (0.90)
26. (d)
 F = 600 - 2 ¥ 105 t = 0 fi t = 3 ¥ 10 -3 sec
p(t ) = A Èiˆ cos (kt ) - ˆj sin (kt )˘

Î ˚ -3
Impulse I = t F dt = 3¥10 (600 - 2 ¥ 103 t ) dt
 d p Ú 0 Ú 0
F= = A ÎÈiˆ( - sin kt ) k - ˆj (cos t ) k )˚˘
dt -3
= [600 t - 10 5 t 2 ] 30 ¥ 10 = 0.9 N ¥ s

   
p ◊ F = 0, i.e. angle between p and F is 90°
32. (10)
27. (90) The acceleration of a rocket is given by

 v Ê Dm ˆ 400 Ê 5 ˆ
Given that p = pxiˆ + p y ˆj = 2 cos t iˆ + 2 sin t ˆj a= -g= - 10
m ÁË Dt ˜¯ 100 ÁË 1 ˜¯

 dp
\ F= = - 2sin t iˆ + 2 cos t ˆj = (20 - 10) = 10 m/s2
dt
    33. (25)
Now, F · p = 0 i.e. angle between F and p is 90°.
When all are pulling
28. (417) 
Fnet = 100 ¥ 3 iˆ  ...(1)
u = 100 m/s, v = 0, s = 0.06 m
when A stops
u2 (100)2 1 ¥ 106  
Retardation = a = = = Fnet - FA = 100 ¥ 1(- iˆ)  ...(2)
2 s 2 ¥ 0.06 12
when B stops
5 ¥ 10 -3 ¥ 1 ¥ 106 5000  
\ Force = ma = = = 417 N Fnet - FB = 100 ¥ 24 ˆj
12 12  ...(3)
 
29. (0.12) from these three get FA + FB = (700 iˆ - 2400 ˆj ) N
Velocity by which the ball hits the bat
hence acceleration of the cart
  
v1 = 2 gh1 = 2 ¥ 10 ¥ 5 or v1 = +10 m/s = 10 m/s  FA + FB (700iˆ - 2400 ˆj )
a= = m/s 2
Velocity of rebound m 100
  
v2 = 2 gh2 = 2 ¥ 10 ¥ 20 = 20 m/s or v2 = -20 m/s a = (7iˆ - 24 ˆj ) m/s 2 fi a = 7 2 + 242 = 25 m/s 2

4 Solutions

34. (4) d
p Mu Ê M ˆ Ê Lˆ Ê Lˆ
pd =Á <
Dv = 2v; Dt = 2 = M + m Ë M + m ˜¯ ÁË t0 ˜¯ ÁË t0 ˜¯
v 2v
(i) is O.K.
Dv m ( 2v ) 4mv 2
F =m¥ = = Ê mu ˆ Ê m ˆ Ê Lˆ
Dt d pd
p v = Á =
2v Ë m + M ˜¯ ÁË m + M ˜¯ ÁË t0 ˜¯
35. (1) (ii) is O.K.
  
Change in momentum = DP = Pf - Pi 1 1
K.E. of the system = Mv 2 + m(u - v)2
   2 2
fi Pf = Pi + DP 2
1 Ê mM ˆ Ê L ˆ
È8 ˘ =

fi mv f = (mv0 ) iˆ + Í Fdt ˙ ˆj 2 ÁË m + M ˜¯ ÁË t0 ˜¯
Í Ú
˙ (iv) is wrong.
   Î0 ˚
 2 È1 ˘  38. (d)
v f = 6iˆ + ¥ 1 ¥ 4˙ ˆj fi v f = 6iˆ + 8 ˆj Consider a system of two particles.
0.5 ÍÎ 2 ˚
If linear momentum of the system is zero, this does not implies that
\ n = 1 momentum of each particle is zero.
36. (65) Let say the two particles, having equal magnitude of momentum, are
Let mass of the particle be m. Then moving in opposite direction. Thus magnitude of linear momentum
of each particle is a finite quantity i.e. |p1| = |p2| π 0.
         2 2
F1 + F2 + F3 + F4 + F5 F + F3 + F4 + F5 p1 p2
= 4iˆ; 2 = 7 ˆj Thus kinetic energy of the system KE = + π0
m m 2m1 2m2
 
F1 F Hence statement 1 does not imply 2.
+ 7 ˆj = 4iˆ fi 1 = 4iˆ - 7 ˆj
m m If kinetic energy of the system is zero, this implies that momentum
  of each particle is zero i.e. p1 = p2 = 0
F1  F1
= Acceleration due to F1 fi = 44 + 7 2 = 65 m/s 2 Thus linear momentum of the system p = p1 + p2 = 0
m m
Thus statement 2 implies 1.

39. (d)
Conservation of Momentum
37. (a)

By the momentum conservation

3m 4v

mv = v1 fi v1 =
4 3
40. (d)
0 v1 v2
Velocity of the man with respect to the plank is
+
L
u= m1 m2
t0
When the man moves to the left, the plank moves to the right. 0 = –m1v1 + m2v2 fi m1v1 = m2v2 = p
Velocity of the plank w.r.t. the ground = v
Velocity of the man w.r.t. the ground = u – v 1 1 E p2 /2 m m2
= 2 =
By the momentum conservation E2 p /2 m2 m1
mu 41. (d)
m(u – v) = Mv fi v =
M+m Velocity of the body after 5 s,
Velocity of the man w.r.t. the ground = u – v v0 = 100 – g ¥ 5 = 50 m/s
Newton's Laws of Motion 5

v 30 kg
45. (d)
12 v
+ y 0
150 m/s 20 kg 12
x
q
 5m m m 3m
50 m/s 50 kg By the momentum conservation
x: 5 m ¥ 0 = m ¥ 12 + 3m v cos q fi v cos q = – 4
y the conservation of momentum
B
y: 5m ¥ 0 = m ¥ 12 + 3m v sin q fi v sin q = – 4
50 ¥ 50 = 20 ¥ 150 + 30 V
The third piece will move
50 4
v= -
m/s q
3
v
42. (a)
4 v = 4 2 m/s, q = 45°

omentum conservation at the highest point


M
2mv cos q = –mv cos q + mv ¢
v¢ = 3v cos q
43. (c) 46. (a)
mm 12 m/s
10 10
0
q q
y 8 m/s
(3 + m) 1 kg 2 kg m
v = 4 m/s
x x
Resultant momentum of two parts is

q + q = 90° fi q = 45°
x 10 t 2 (1 ¥ 12)2 + (2 ¥ 8)2 = 20 kg m/s

tan q = = = 20 = m ¥ 4 fi m = 5 kg
y 1 t
gt 2
2 47. (b)
2 Let the momentum of the third piece be px iˆ + py ˆj. By the
tan 45° = fi t = 2 s momentum conservation
t
44. (b) 0 = - 3 piˆ - 4 pjˆ + px iˆ + py ˆj
40 m/s
y 10 m/s   = (- 3 p + px )iˆ + (- 4 p + py ) ˆj
–3p + px = 0 fi px = 3p
x
q –4p + py = 0 fi py = 4p
3m m 2m
A B The momentum of the third piece
v
By the momentum conservation 4p
P
x: 3m ¥ 10 = 2mv cos q
fi v cos q = 15
y: 0 = m ¥ 40 + 2mv sin q q
3p
fi v sin q = –20

v= (v sin q )2 + (v cos q )2 = 25 m/s P = (3 p)2 + (4 p)2 = 5 p
6 Solutions

4p Ê 4ˆ 3d
tan q = fi q = tan -1 Á ˜ 3md = m ¥ 0 + 2m x0 fi x0 =
3p Ë 3¯ 2
3d d
48. (d)
x¢ = x0 – d = -d=
2 2
40
10 10 v (i) is O.K.
a Momentum conservation at the highest point is
+
3m m m m
3m v cos q = –mv cos q + 2mv ¢ fi v¢= 2v cos q
20 m/s
1 2
K 2 m 2 ¥ 2 m(2 v cos q )
= =8
60° Km 1 2
¥m v
( cos q )
3m 2
elocity of shell at the highest point = 20 cos 60° = 10 m/s
V (ii) is O.K.
Momentum conservation at the highest point K.E. before explosion
x: 3m ¥ 10 = –m ¥ 10 + m v cos a fi v cos a = 40 1 3m 2
y: 0 = m ¥ 40 + m v sin a fi v sin a = – 40 Ki = ¥ 3m (v cos q )2 =
v cos2 q
2 2
Third piece:
K.E. after explosion
40
a
1 1
Kf = m( v cos q )2 +
¥ 2 m (2 v cos q )2
2 2
v
40 9m v 2 cos2 q
  =
v = 40 2 m/s 2
a = 45° Ê Kf ˆ

Á - 1˜ ¥ 100 = 200%
49. (c) Ë Ki ¯
2m m
u=0
20m
A v2 20 m/s (iii) is O.K.
B B Both parts will strike the ground simultaneously, (iv) is wrong.
80m
60m 51. (b)
80m 2m m By spitting horizontally in the direction opposite to the desired
C x2 x1 direction of his motion. By doing so, he is giving some momentum
to the lost mass (spit). So to conserve momentum, he himself gets
The center of mass will follow the vertical straight line. The time some momentum in the opposite direction.
2 ¥ 80 52. (a)
taken by ball to fall 80 m, t1 = = 4 s and the time taken to
10 Momentum of bullet = momentum of rifle = p

2 ¥ 20 p2
fall by 20 m, t2 = = 2 s, hence the time taken by the ball
Kbullet = , m: mass of bullet
10 2m
(or two pieces) from B to the ground, t0 = t1 – t2 = 2 s. p2
At B, the momentum conservation in horizontal direction:
Krifle = , M: mass of rifle
2M
0 = m ¥ 20 – 2 m v2
fi v2 = 10 m/s, x1 = 20 t0 = 40 m, x2 = v2 t0 = 20 m Since M > m, Krifle < Kbullet
Separation between blocks = x1 + x2 = 60 m. 53. (b)
50. (b)
v0

v
Smooth floor

v0: vB/P: velocity of boy relative to platform


v: vp/g: velocity of platform relative to ground
vB/g = v0 – v
By the momentum conservation
Newton's Laws of Motion 7

0 = m(v0 – v) – Mv 59. (100)


mv0
v=
M+m
54. (3.20)
If the man starts walking on the trolley in the forward direction
then whole system will move in backward direction with same
momentum. 0.2 u
Momentum of man in forward direction = Momentum of system 0 = – 4v + 0.2u fi v =
(man + trolley) in backward direction
4
1 1
fi 80 ¥ 1 = (80 + 320) ¥ v fi v = 0.2 m/s ¥ 4v 2 + (0.2)u 2 = 1.05 ¥ 1000
2 2
So the velocity of man w.r.t. ground 1.0 – 0.2 = 0.8 m/s 2
Ê 0.2 u ˆ
\ Displacement of man w.r.t. ground = 0.8 × 4 = 3.2 m 2Á + 0.1 u 2 = 0.105 u 2 = 1.05 ¥ 1000
Ë 4 ˜¯
55. (2.50) u = 100 m/s
M
Momentum of one piece = ¥3 60. (5)
4 0 200 m/s v
M
Momentum of the other piece = ¥4 +
4

9M 2 5M
\ Resultant momentum = +M2 = Gun Bullet Gun
16 4
The third piece should also have the same momentum. Let its Since for (gun + bullet) the system, the external force is zero. Hence
velocity be v, then by momentum conservation
Initial momentum = Final momentum
5M M 5
= ¥ v or v = = 2.5 m/sec 0 = mu + Mv
4 2 2 Mass of bullet, m = 50 g = 50 ¥ 10–3 kg, u = 200 m/s
56. (216) Mass of gun, M = 2 kg
0 6 m/s v 0 = 50 ¥ 10–3 ¥ 200 + 2 v
v = –5 m/s
+
where the –ve sign indicates that the velocity of gun is opposite to
20 kg 2m 3m that of the bullet.
2m + 3m = 20 fi m = 4 kg 61. (3)
By the momentum conservation 0 v¢
0 = 2m ¥ 6 – 3 mv v
fi v = 6 m/s
1 1
¥ 3 m ¥ v 2 = (3 ¥ 4)(6) 2 = 216 J
2 2
1 2
4m
57. (4.80)
0 80 m/s v
By momentum conservation,
+ v
0 = mv + 3mv¢  fi  v¢ = -
3 kg 1 kg 2 kg
3
1 2
3 ¥ 0 = 1 ¥ 80 – 2v Given K1 = mv = K
2
fi v = 40 m/s
1
K2 = ¥ 3mv¢ 2 = K ¢
Kf = 1 ¥ 1 ¥ (80) 2 + 1 ¥ 2 ¥ (40) 2
2
2 2
= 4800 J = 4.8 kJ K¢ Ê 1ˆ
2
1
= 3 ( v ¢ /v ) 2 = 3 Á ˜ =
58. (1.92) K Ë 3¯ 3

3 × 1.6 + 6 × V = 0 fi v = 0.8 m/s K
1 K¢ =
KE = × 6 ×0.8 × 0.8 = 1.92 J 3
2
8 Solutions

OR 64. (7)
0 p Total mass 5 m is split in a ratio 1 : 1 : 3, hence masses of three
p
pieces are m1 = m, m2 = m and m3 = 3 m.

4m m 3m

Due to the momentum conservation, momentum of two pieces is


equal and opposite. Let the velocity of heavier piece is v at an angle q. Here, we have to
2 2
conserve momentum in the x- and y-direction separately.
K = p , K ¢ = p By momentum conservation
2m 2.3m x-direction: 5m ¥ 0 = mv0 + 3mv cos q
K¢ 1 K v0
= fi K ¢ = v cos q = -
K 3 3 3
where K¢ is the K.E. of larger fragment. y-direction: 5m ¥ 0 = mv0 + 3mv sin q
62. (0.50) v
v sin q = - 0
When the smaller block is moving on the vertical part, the (m + M) 3
system will move with velocity v0 in horizontal direction. The –ve sign indicates that the velocity is in opposite direction

m
M
v0

v0
v = 2, q = 45∞
Since no external force in horizontal direction, hence by the 3
momentum conservation
mv = (m + M)v0
mv 50 ¥ 20 1
v0 = = = m/s
m + M (50 + 1950) 2

63. (115)
Mass number = sum of the number of protons and neutrons
A=Z+N
Z: number of protons
N: number of neutrons
The direction of the third piece is at an angle 135° from either of
Let mp  mN = m smaller pieces.
U238: mass: 238m Initial K.E. = 0
Th234: mass: 234m 2
Ê ˆ
a-particle (2He4): mass: 4m Final K.E. = 1 mv02 + 1 mv02 + 1 ¥ 3m Á 2 v0 ˜
p 2 2 2 Ë 3 ¯
p
0
v0 v 1 2
= mv02 + mv0
3
4
= mv02
238m 4m 234m 3
By momentum conservation 4
K.E. released in explosion = mv02
0 = 4mv0 – 234mv 3
4v0 2v0 OR
v= =
234 117 This problem can be solved without conserving momentum
v: Recoil speed of Th234 conservation in x- and y-direction separately.
Newton's Laws of Motion 9

Since the bomb is at rest, hence initial momentum is zero and hence F2 = dp2/dt…(ii)
the final momentum of system of pieces will be zero, due to the Adding (i) and (ii), we get
momentum conservation. F1 + F2 = dp1/dt + dp2/dt
The resultant momentum of smaller pieces will be equal and Since the system is isolated, so the total change in momentum or
opposite to the momentum of heavier piece. rate of change of momentum (dp1/dt + dp2/dt) should be zero. Thus,
F1 + F2 = 0 or F1 = –F2
This is the statement of third law.
68. (b)
When a person jumps from a boat, his leg pushes the boat backwards.
Let m v0 = p In turn, the boat pushes him in the forward direction as per Newton’s
third law.
69. (c)
When the man applies force on the water surface, then according to
Resultant momentum P = 2 p Newton’s third law, a reaction force will act on the him (or ice) to
q = 45° move him to the shore.
Hence the momentum of heavier piece is equal and opposite to P. 70. (a)
Newton’s law is equivalent to law of conservation of linear
momentum. This can be proved as below:
Suppose an isolated system of two bodies A and B. Suppose A
exerts force F1 on B and B exerts F2 on A. Now, rate of change of
momentum of A is dp1/dt and that of B is dp2/dt. Therefore, from
second law,
F1 = dp1/dt…(i)
P= 2 p = 2 mv0 F2 = dp2/dt…(ii)
Adding (i) and (ii), we get
P = 3mv
F1 + F2 = dp1/dt + dp2/dt
3mv = 2mv0 Since the system is isolated, so the total change in momentum or
rate of change of momentum (dp1/dt + dp2/dt) should be zero. Thus,
2 v0 F1 + F2 = 0 or F1 = –F2
v=
3 This is the statement of third law.
v: Velocity of heavier piece. 71. (c)
u = velocity of bullet
Third Law of Motion (Using Free Body Diagrams) dm
= Mass thrown per second by the machine gun
65. (c) dt
= Mass of bullet ¥ Number of bullet fired per second
According to Newton’s third law, action-reaction pairs are always
equal. = 10 g ¥ 10 bullet/sec = 100 g/sec = 0.1 kg/sec
udm
66. (a) \ Thrust = = 500 ¥ 0.1 = 50 N
Gravity and normal reaction of the head act on the box. Thus, one dt
action-reaction pair acts on the box. 72. (d)
67. (c) Components of accelerations of A and B along normal (nˆ ) to the
We can derive Newton’s third and first laws from second law. incline of wedge should be same.
If force on particle is zero, then according to Newton’s second law
F = ma
=> 0 = ma
=> a = 0 (since mass cannot be zero)
So, this shows that the body moves with constant velocity or remains
at rest if no external force on the system. This is the statement of first
law itself.
Suppose an isolated system of two bodies A and B. Suppose A
exerts force F1 on B and B exerts F2 on A. Now, rate of change of
momentum of A is dp1/dt and that of B is dp2/dt. Therefore, from
second law,
fi A sin q = a cos (2q - 90∞)
F1 = dp1/dt....(i)
10 Solutions

fi A sin q = a sin 2 q The FBDs of A and B are:


fi A = 2 a cosq
73. (a)
Force on B = m2a2  \  Force on A = m1a1
m2 a2
Acceleration of A =
m1

74. (a)
N
Total mass of bullets = Nm, time t =
n
Momentum of the bullets striking the wall = Nmv For the horizontal equilibrium of the system (A and B):
Nmv NAW = NBW ...(1)
Rate of change of momentum (Force) = = nmv and by diagrams:
t
6
75. (c) cos q = fi q = 53°
 10
Fgravitational acts downwards.
Applying NLM on sphere B

Fdrag acts opposite to the direction of motion. fi NAB sin q = w ...(2)
 w 3 3w
Now let us see how Frotor acts. and N BW = N AB cos q = ¥ =
(4/5) 5 4
3w
So N AW = N BW =
4
78. (a)
F.B.D. of block
N

(i) (ii) F = 10 N

Wings of helicopter will push the air perpendicular to their plane of mg = 10 N


rotation as shown in figure (i). From third law, air will apply force N2 = F2 + (mg)2

Frotor on the wings as shown in figure (ii). {as there is no acceleration in the direction of N}
There is no force ma. Basically ma is the resultant of all the forces N = 10 2 N
acting on a body. Moreover, here a is zero because velocity of
helicopter is constant. 79. (c)
76. (c) F.B.D. of block A
N
F
Acceleration of all blocks in both cases is the same and is a =
m+M g
In case I : required normal reaction RI is equal to net force on block mAg
of mass m
Applying Newton’s second law for block A in vertical direction
\ RI = ma
mAg – N cos q = mA g
In case II : required normal reaction RII is equal to net force on block
of mass M. where q is the angle of the wedge
\ RII = Ma fi N cos q = 0
Hence, RII > RI as q < 90°    \  N = 0

77. (c) 80. (b)


From Newton’s third law a force F acts on the block in forward
direction.
A
v0
m M

l
Newton's Laws of Motion 11

F 84. (2)
Acceleration of block a1 =
M
F
Retardation of bullet a2 = Here, each block moves same distance in equal time, hence their
m
acceleration is same.
Relative retardation of bullet
a
F ( M + m)
ar = a1 + a2 =
Mm F0 m 2m F0
Applying v2 = u2 −2ar F0 = (m + 2m) a fi a =
3m
2 F ( M + m)
0 = v02 - ◊ a
Mm a F0 2 F0
Therefore, minimum value of v0 is F0 N   N N = 2ma = 2m ¥ = = 2m / s 2
m 2m 3m 3
2 F ( M + m )
or v0 =
Mm 85. (1.60)
a
81. (3)
u = velocity of bullet
dm 30 N 2 kg 3 kg 5 kg
= Mass fired per second by the gun
dt
30 = (2 + 3 + 5) a fi a = 3 m/s2
dm
= Mass of bullet (mB) ¥ Bullets fired per sec (N) a
dt a a
Ê dm ˆ N1
Maximum force that man can exert F = u Á N2 5 kg
Ë dt ˜¯ 30 N 2 kg N1 3 kg N2

\ F = u ¥ mB ¥ N
30 – N1 = 2a N2 = 5a = 5 ¥ 3
F 144
fi N= = =3 = 2 ¥ 3
= 15 N
mB ¥ u 40 ¥ 103 ¥ 1200
N1 = 24 N
82. (12)
86. (3)
m a0

N1
mg

mg 3mg
mg – N1 = m a0 = fi N1 =
4 4
N1

2m a0
9 cos a = v sin a ...(i)
19 - R N2
= tan a  ...(ii) 2mg
12
(R + 5)2 = (12)2 + (19 – R)2 g mg
2mg + N1 – N2 = 2m a0 = 2m ¥ =
fi R = 10 4 2
Hence, from (i) and (ii) 3mg mg
v = 12 m/s 2mg +
4
- N2 =
2
83. (25) 9mg
N2 =
Let us find vertical force: 4
F1 sin 30∞ + F2 sin 60∞ OR

1 Ê 3ˆ
= 20 ¥ + 10 3 Á ˜ = 25 N
2 Ë 2 ¯

This is greater than mg. So contact of body with surface will be lost
and reaction will be zero.
12 Solutions

g 91. (64)
(m + 2m)g – N2 = (m + 2m) a0 = 3m ¥
4 60 - (18 + 30)
a= = 2 m/s 2
9mg 6
N2 =
4
0N
87. (4) =6
NA F 1

N 5 kg
= 18
FBD of A : F 2


sin 3
0.5g g
6
0.5g – NA = (0.5) (2)
So, all the three blocks will accelerate upwards with
5 – NA = 1 a = 2 m/s2. N1
NA = 4 N  N1 (18 + 5) = 1 ´ 2 30°
1g sin 1 kg
88. (40)  N1 = 25 N
Mass of system = 2 + 4 + 6 = 12 kg 18 N
F2 =
Now, 24 – 12 = 12 a
fi a = 1 m/s2  N2 – (18 + 15) = 3 ´ 2 N2

  System 2 kg System 6 kg  N2 = 39 N 30°


39 sin (2 + 1) kg
  24 – f1= 2(a) f2 – 12 = 6(a)
  f1 = 22 N f2 = 18 N 18 N
F2 =

89. (50)
T 92. (3)
C
2 N sin q = mg
B
mg mg R
N= cosecq =
53° 37° 2 2 l2
2
R - R
4
N N
30 N 40 N θ l/2 l/2
When l = R,
100 N mg

mg R mg
N= =
Resultant of 30 N and 40 N is 50 N in vertical direction. 2 2 3
3R
Hence, T = 50 N
4
90. (45)

N´ What's with all the Tension in the Room?


37° 93. (d)
37° N´
N 60 Weight of lift = Mg
Maximum tension = nMg
N nMg - Mg
\ Maximum acceleration =
60 N M
= (n − 1)g
N 3 and maximum retardation = g
From FBD of system “sphere + block”, =
60 4
corresponding velocity–time graph for shortest time will be as
N = 45 N follows:
Newton's Laws of Motion 13

a=0
N0

mg

vm vm T – N0 – mg = 0 ...(ii)
Here (n - 1)g = or t1 =  ...(1)
t1 (n - 1) g Solving (i) and (ii), we get

vm v ( M + m)g ( M - m)g
and g= or t2 = m  ...(2) T = , N0 =
t2 g 2 2
Area under v−t graph is total displacement h. T: force exerted by the boy.
N: contact force between the boy and the box.
1 Note: Whenever bodies are in contact, always transfer normal
Hence h= (t1 + t2 ) vm  ...(3)
2 reaction, not weight.
From (1), (2) and (3) we get, 96. (a)
Boy: T + N0 – Mg = Ma
Ê n - 1ˆ
vm = 2 gh Á fi T + N0 = M(g + a) = 80(10 + 5) = 1200 ...(i)
Ë n ˜¯ Box: T – N0 – mg = ma

fi T – N0 = m(g + a) = 40(10 + 5) = 600 ...(ii)
94. (a) T = 900 N, N0 = 300 N
97. (b)
sin q = r/R fi q = 30°

T dF cos q dF
q
m dx dF sin q
dx

T d mg

FBD of sphere: Take a small element of the rope subtending angle da at the center
of rope.
W = T sin q
mda
Mass of this small element: dm =
3 2p
and cos q =   fi  q = 53°
5 dF sin q = 2T sin(da/2) fi dFsin q = Tda
W 5W mda
fi T= = dF cos q = dmg fi dF cos q = g
sin 53∞ 4 2p

95. (b) Dividing:

Boy: T 2p mg tan q W W
tan q = fi T = = =
T
mg 2p 2p 3 p 12

98. (b)
a=0

N0
Mg

T + N0 – Mg = 0 ...(i)
Box:
14 Solutions

Reading of the weighing machine = 2T + weight of the machine. 103. (b)


As weight of the machine is constant. Wall
2m1m2 A v
T= A
m1 + m2 co
vA s
60
So reading is maximum for the case m1m2 is maximum as °
60°
m1 + m2 in all cases is same.
99. (b)
T = N + 30 g ...(i)
T + N = 50 g (ii) B vB = vC = 3 m/s
30°
30°
T Floor

30 g
N vB = 3 m/s
50 g
N v A cos60∞ = vB cos30∞ fi v A = 3 m/s

\ N = 10 g = 100 N
104. (d)
100. (a)
v1 v2
a+0 a
Let a = accn of m1 accn of pulley = = 1 2v1 2
2 2
If acceleration of m2 = b
2v1 v2
b a
Then 0 + =
2 2
2v1 + v2
Hence, a = b = v3
2
T = m1 a, m2g – T = m2 a
3 2v1 + v2 = 2v3
m2 g v3
\ a=
m1 + m2

101. (c) 105. (d)


F.B.D of the mass M:
60°
os
vB c
vB 30°
B
vA = 1 m/s
T=F
2F 60°
ma = 2T = 2F fi a =
m
vA cos 60°
4F
So, acceleration of point P is 2a = (from constraint relation)
m VA cos 600 = VB cos 600 fi VA = VB
102. (c)
Forces on the rod R1 are shown in the figure: 106. (a)
dx1 dx2
x1 + x2 = l fi + =0
dt dt
vP – vB + vP = 0

x2
P ap
x1
aB m

aP 1
FNet = 2 2 F 2vP = vB. It gives 2 aP = aB fi =
aB 2
Newton's Laws of Motion 15

107. (c)
For B: mg – T = ma Q

aC aD
C D
2T a2
Puley Q
m A
aD - aC
mg T aQ =
2
a mB aD - 2
4= fi aD = 10 m/s2 (Ø)
mg 2
111. (c)
aA aB
ma aA
For A: 2T - mg = T
2
A 2m T
5 T¢
fi mg = ma O
2 T
2g
a= B m aB
5
108. (a) mg

v v v aB + 0
aA =
m1 m1 m1 2
2aA = aB
maB = m(2aA) ...(i)
m2 v1 2T = 2m·aA ...(ii)
m2 v3
m2 v2 2 (i) + (ii)
2 mg = 6 m aA
v1 = v v2 = v/2 v3 = 2v
2 mg
T ¢ = 2 ma A =
v1 : v2 : v3 = 1 : 0.5 : 2 = 2 : 1 : 4 3
109. (d) 112. (c)
From pulley (3)

1 O O 2 O

a3 2a1
O
m3 2a1
a3
3
a2 Let in a small time dt, displacement of A is dx and displacement of
a2 B is dy. Increase in length l1 = dx.
a2 a1
m2 m1 Increase in length  2 = dx sin q - dy cosq
But net increase in length should be zero, so
2 a1 - a3
a2 = dx + dx sin q - dy cosq = 0
2
2 a2 = 2 a1 - a3 (1 + sin q ) dx
fi dy =
cosq
2 a2 - 2 a1 + a3 = 0
dy Ê 1 + sin q ˆ dx Ê 1 + sin q ˆ
fi =Á ˜ fi v = Á u
110. (c) dt Ë cosq ¯ dt Ë cosq ˜¯
Acceleration of pulley P
113. (d)
a A + aB 1 + 7
aP = = = 4 m/s2 (upward) If system is released, the wedge will slide towards right. Let wedge
2 2 slide towards right by distance x. The string of length 3x will be
Acceleration pulley Q will be 4 m/s2 downwards. adjusted on the inclined face.
16 Solutions

Let dwedge = x (towards right) The total force on the block is:
Hence dblock, wedge = 3x The x-component = 18 N
The y-component = 24 – mg = 24 – 20 = 4 N
The x-component of the acceleration = 18 N/2 kg = 9 m/s2
The y-component of the acceleration = 4 N/2 kg = 2 m/s2
117. (a)
  
dblock = dblock, wedge + dwedge For movable pulley

= (3 x )2 + x 2 + 2(3 x )( x )cos(180 - q )

= 10 x 2 - 6 x 2 cosq = x 10 - 6 cosq

4 26
= x 10 - 6 ¥ =x O + aA
5 5 aB =
2
Hence, acceleration of block aA = 2aB ...(i)
 26 Free body diagrams of A and B:
am = A = 26 m/s2
5

114. (c)
The F.B.D. of section of rope between A and B having acceleration
a towards left is
Equation of motion of blocks A and B
mAg
For A: T- = m A a A = m A (2 aB )  ...(ii)
2

Applying Newton’s second law on section AB and section AC of For B: mB g - 2T = mB aB  ...(iii)


rope we get From (ii) and (iii), [2(ii) + (iii)]
M
TB – TA = Ma and TC – TA = a mB g - m A g = ÈÎ4 m A + mB ˘˚ aB
5
T + 4TA [ mB - m A ] g [4 - 2] ¥ 10
Solving TC = B fi aB = =
5 [4 m A + mB ] [4 ¥ 2 + 4]
115. (d)
20 5
= = m/s2
12 3
10
Hence, a A = 2 aB = m/s2
3
118. (c)
As no friction is involved, the tensions in the segments AC and AE
of the string must be the same. Let its magnitude be T. For the ring
For student A to just lift off the floor, tension T in string must be A to be at rest on the smooth loop, the resultant force on it must be
greater than or equal to 700 N. along AO, O being the center of the loop; otherwise there would be
The F.B.D. of student B is a component tangential to the loop. Hence
∠OAE = ∠OAC = ∠AOE = 30°.
Applying Newton’s second law
The same argument applies to the segments BD and BE.
T – mg = ma fi 700 – 600 = 6 a Then by symmetry the point E at which the string carries the third
5 weight must be on the radius HO, H being the highest point of the
or a = m/s 2 loop, and the tensions in the segments BD and BE are also T.
3
Now consider the point E. Each of the three forces acting on it,
116. (a) which are in equilibrium, are at an angle of 120° to the adjacent
The magnitude of the force (from the string) is T = 30 N one. As two of the forces have magnitude T, the third force must
The x-component = T sin q = 30 ¥ 3/5 = 18 N also have magnitude T. Therefore, the three weights carried by the
The y-component = T cos q = 30 ¥ 4/5 = 24 N string are equal.
Newton's Laws of Motion 17

119. (d) l1 + l2 + l3 = constant


FBD of C
mg – 2T = mc ...(i) d1 d 2 d 3
\ + + =0
If we assume acceleration of a to be ‘a’ upwards; dt dt dt
by constraint equation we get:
–a – a + c + a + c = 0 fi a = 2c (– 10) + (– 5 – 10 cos 37°) + (– 5 sin 37° + vy) = 0
and for block A: vy = 26 m/s.
2T = mA.a but as mA = 0 fi T = 0 Horizontal component of velocity is 14 m/sec and vertical
component of velocity is 26 m/sec.
so we get C = g and a = 2g≠.
122. (1.50)
120. (a)
B A
Initially acceleration of B and A is same along the
string which is given by
2 mg sin 30∞ - mg sin 30∞ g
a= = P1
3m 6
After 2 seconds, speed of both A and B is P2

g 10
VB = VA = ¥ 2 = m/s m
6 3
Now, VA = 2 m/s (towards right)
When B is caught for a moment and released again, speed of B
becomes zero, VA
VP1 = = 1 m/s   (upwards)
10 2
while A is still having a speed m/s up the inclined due to which VB = 2 m/s (towards left)
3
string will become slack. But A will decelerate and B will accelerate.
Because of this the string will become tight again after A and B Now 2VP2 = VB + VP1
travel the same distance.
VB + VP1 2 +1
Let this time interval be t VP2 = = = 1.5 m/s
2 2
10 1 1 10 g
¥ t - g sin 30∞ ¥ t 2 = g sin 30∞ ¥ t 2 fi = t 123. (75)
3 2 2 3 2
= t; as t ≠ 0 v 2 25
a= = = 2.5 m/s 2
2 s 10
2
t= sec
3
At this time speed of A is given by
10 g 10 10 2
V= - ¥t = - ¥ =0
3 2 3 2 3

121. (b) For 6 kg: – F – 2T = 6a ...(1)


For 2 kg: – T – 2g = 2 (2a) ...(2)
37°
10 m/s From (1) and (2), F = 75 N
5
m 37° l3 l2
A /s
B
vx 124. (0)
C
vA = 10 m/s
vy l1
37°

Let vx and vy be the horizontal and vertical component of velocity


of block C. Let c be acceleration of wedge C.
The component of relative velocity of B and C normal to the surface a be acceleration of block A w.r.t. wedge c.
of contact is zero. b be acceleration of block B w.r.t. wedge c.
\ 10 + 5 cos 37° – vx = 0 ...(1) Applying Newton’s law in horizontal direction to system of
vx = 14 m/s A + B + C.
from the figure mc + m (c – a cos 37º) + m (c + b cos 53º) = 0 ...(1)
18 Solutions

Applying Newton’s law to block A and B along the incline gives. 1 2


In case of A: mg sin37º = m (a – c cos 37º) ...(2) s = ut +
at
2
In case of B: mg sin53º = m (b + c cos 53º) ...(3) 1 6g 2
6= ¥ ¥ t fi t2 = 1 fi t = 1 s (∵ u = 0)
Solving (1), (2), and (3), we get c = 0 2 5
125. (5) 129. (8)

100 - 40 100 - 50 a
aA = = 15 m/s 2 , aB = = 10 m/s 2
4 5 M
2 1
fi a A - aB = 15 - 10 = 5 m/s
2T
126. (15) a
T = m2g and 8T + 4T + 2T + T = m1g 2
fi 15T = m1g T T
m2
fi 15 m2g = m1g m1
fi m1 = 15m2
m1 = 1 kg m2 = 2 kg
g
am1 =
3
8T 8T
m1 will move with constant velocity if a = 0.
g
So, downward acceleration should be .
4T 4T 3

4g
2T 2T fi T =
3

g 8g
T \ M = fi M = 8 kg
T 3 3
m1 m2 130. (0) N1

127. (5) m1 = 20 kg
T
m2 = 30 kg
T T
θ θ θ T = m1 am1 m1g
θ θ θ
T´ N2
W fi 20 am1 = 180 N1

2T sin q = W ... (i) fi  am1 = 9 m/s 2


T cos q = T¢ ... (ii) am2 = 0
m2g
W cos q W
From (i) and (ii), T ¢ = = cot q = 5 N 131. (2)
2 sin q 2
For block C, we have
128. (1) 2T – 8g = 8a ... (i)
4g – T = 4a For block A, we have
T–g=a 5g – T = 5(2a) ... (ii)
3g = 5a From (i) and (ii),
a = g/14
3g
fi  a = So, we have, aA = g/7
5
aC = g/14
6g 132. (6)
Relative acceleration =
5 T1 sin 53° + T2 cos q = 10
By second equation of motion, T1 cos 53° = T2 sin q
Newton's Laws of Motion 19

Ê T sin q ˆ T1 136. (48)


sin53° Á 2 + T2 cos q = 10
Ë cos 53∞ ˜¯ θ
T2 T sin q = mg sin 2q
53° sin 2θ θ
30 T = mg
T2 = sin θ
3 sin q + 4 cos q T θ 2θ
  = 2mg cos q T sinθ
10 N
30
T2 min = =6N   = 48 N
5 2θ
mg
mg sin2θ
133. (2)
137. (35)
FBD of the blocks A, B and C:
Mass 2 kg is at rest, so T2 = 20 N
2T T T So for (2),
T2 – mg = ma
T1 T1
20 – 10 = 1 ´ a (1)
A B C
a = 10 m/s2 m
T2
T2
T1 = 2 T2 = 40 N (2)
m
40 30 30 So for (1),
20
T2 – mg = ma
Conditions of forces on B and C are same, so they both
will move with same downward acceleration a. The block 40 – 10 = 1 ´ a
A moves with the same acceleration a upwards. a = 30 m/s2
For block A : 2T – 40 = 4a ... (i) End of rope is coming down with 5 m/s2. So, acceleration of
For block B or C : 30 – T = 3a ... (ii) (1) w.r.t. his rope is 30 + 5 = 35 m/s2 upwards.
From above two equations, a = 2 m/s 2 Acceleration of monkey (2) with respect to his rope is zero
because he is just holding the string.
134. (2)
138. (2)
T
T (0.5) – 2TvP – T × (1) = 0
T 2vP = –0.5
T 2 \ 8 |vP| = 2
a = 2 m/s
A
N1 139. (180)
B
T T
T=2N vA
1 kg T vB
N1 mg
mg N2 A T
T mg
\ a = 0
fi Relative acceleration = 2 m/s2 (3T)vA – T(vB) = 0
\ vB = 3vA = 180 cm/s
135. (250)
T 140. (13)
2T N
Springs: Playing with a Slinky

141. (a)
As the spring balances are light, so tension in both the springs will
mg mg be same and equal to weight of block suspended.
N
142. (d)
T + N = mg ... (i) and 2T = N + mg ... (ii)
The fictitious force will act downwards. So the reading of spring
From (i) and (ii), balance will increase. In case of physical balance, the fictitious force
T = 250 N will act on both the pans, so the equilibrium is not affected.
20 Solutions

143. (c) 4kx


Let the initial compression of spring be l. Then the acceleration after \ Acceleration of the block =
m
the block travels a distance x is
147. (c)
F – kx = mb and kx = ma

k
a= ( - x )
m
\ The graph of a vs. x is Hence m (b – a) = F – 2kx

144. (c) 148. (b)


For first case tension in spring will be After string is cut, free body diagram of block A gives
Ts = 2mg just after ‘A’ is released.

2mg – mg = ma fi a = g mg g
In second case Ts = mg 2 ma A = 3 mg - 2 mg or a A = =
2m 2
mg Free body diagram of block B gives
maB = mg or aB = g

149. (c)
kxcos53° = mg fi kx = 5 mg/3 …(i)
mg
kx
2mg – mg = 2mb kx cos 53°
b = g/2 53°
a/b = 2 kx sin 53°
145. (b) mg
If m1 and m2 are masses of blocks, then tension T in the string as
well as spring is Just after the string burns; tension in the spring remains same, so no
2m1m2 acceleration in vertical direction.
T= g
m1 + m2 Let a is the acceleration in horizontal direction, then

Kx sin 53° = ma …(ii)
T1 = 2 g, T2 = 2.4 g and T3 = 1.33 g
\ T2 > T1 > T3 From (i) and (ii), a = 4g/3
or x2 > x1 > x3
150. (a)
146. (c) In this case spring force is zero initially
F.B.D. of A and B

Initially the block is at rest under action of force 2T upward and 151. (32.50)
mg downwards. When the block is pulley downwards by x, the If reading of spring balance is T, then applying NLM on (man +
spring extends by 2x. Hence tension T increases by 2kx. Thus the ladder) system
net unbalanced force on block of mass m is 4kx. T – (25 + 5)g = 25 a
Newton's Laws of Motion 21

T – 30 g = 25 a  fi  T – 300 = 25(1) 157. (0.20)


fi T = 325 N = 32.5 kg For 3 kg mass,
T1 3g – kx = 3a ... (i)
152. (30)
T1 –T2 – mg – ma = 0 For 1 kg mass,
T1 – T2 = m(g + a) kx + g – T = a ... (ii)
M
600 = m(10 + 10) For 2 kg mass,
ma
fi m = 30 kg T – 2g = 2a ... (iii)
mg
Adding (i), (ii) and (iii), we get
T2
153. (5) a = (g/3)
5 N force will not produce any tension in spring without support of Hence, x = 0.20 m
other 5 N force. So here the tension in the spring will be 5 N only.
154. (4) Problem Solving Using Newton's Laws
As the mass of 10 kg has acceleration 12 m/s2 therefore it apply 120
158. (a)
N force on mass 20 kg in a backward direction.
N – mg cos 60° = ma cos 60°
\ Net forward force on 20 kg mass = 200 – 120 = 80 N
N = m cos 60° (a + g) = 400 N
155. (20)
Before the string A is cut, let x be elongation in the spring.
a cos 60°
N

60°
a 60°

Reading will be 40 kg.

159. (c)
Impulse = Change in momentum = m(v2 – v1) …(i)
Again impulse = Area between the graph and time axis
As system is in equilibrium.
1 1
Then for lower block, = ¥ 2 ¥ 4 + 2 ¥ 4 + (4 + 2.5) ¥ 0.5 + 2 ¥ 2.5
kx = mg = 20 N 2 2
Just after the string A is cut = 4 + 8 + 1.625 + 5 = 18.625  ...(ii)
a From (i) and (ii), m(v2 – v1) = 18.625
2 kg
18.625 18.625
fi v2 = + v1 = + 5 = 14.25 m/s
kx mg m 2
160. (b)
 
2 kg The sum of forces F1 + F2 = m a
x = 5(10 cos q iˆ + 10 sin q ˆj ) = 30 iˆ + 40 ˆj
For upper block,
  
ma = kx + mg y
\ F1 + F2 + F3 = 0
T2   
2a = 20 + 20 \ F3 = - ( F1 + F2 ) = - 30iˆ - 40 ˆj
a = 20 m/s2 37°
T1
156. (3000) 161. (a)

SFx = 0 fi T2 = 160 N Net force on the particle is zero so that v remains unchanged.
120
160 162. (d)
SFy = 0 fi T1 = 120 N
37° The magnitude of acceleration of hanging 2 kg block is highest
200
Spring force = 120 N in figure B and least in figure A. The magnitude of weight acting
Spring constant, downward on each hanging block (of same masses) is same. Hence
tension shall be highest for block having least acceleration (because
120 N it is tension which decreases acceleration of each block). Therefore
k= = 3000 N/m
4 cm TA > TC > TB.
22 Solutions

163. (d)
The acceleration of block of mass m1 is

m1
a= g = 5 m/s 2 w1 5

m1 + m2 =
w2 4

168. (c)

From FBD of block m1


m1g – T = m1a
fi T = m1 (g – a) = 1 ¥ 5 = 5 N T1 sin q1 = 2 mg
164. (d) T2 sin q2 = mg
F.B.D. of block T1 cos q1 = T2 cos q2
2T = 2F = 4mg 2 mg cot q1 = mg cot q2
fi tan q1 = 2 tan q2
169. (b)
mg F.B.D. of man and platform

fi 2T – mg = ma
a=3g
165. (c)
     
AB = W · BC = T1 · CA = T2
  
AB = BC = CA = 0 (as the block is at rest)
2T + N = 25 g N + T = 50 g
3T = 750 T = 250 N
170. (d)

In a position as shown
2T cos q = W
166. (d)
W
Range of resultant of F1 and F2 varies between (3 + 5) = 8 N and T=
(5 – 3) = 2 N. It means for some value of angle (q), resultant 6 can 2 cos q
be obtained. So, the resultant of 3 N, 5 N and 6 N may be zero and For AB to be horizontal, q = 90°
the forces may be in equilibrium.
TÆ•
167. (a)
Resolving forces at point A along string AB 171. (d)
w1 cos 37° = w2 FBD of 18 kg block
Newton's Laws of Motion 23

T = W1 W1
q q

W2

1
fi 2 ¥ 100 cosq = 100 fi cosq = fi q = 60∞ = 2q = 120∞
fi T = 180 N   {as it is at rest} 2
For the lower pulley: 175. (c)
C 120° F

T1 B T2
120° 120°
T = 10 N

A
By drawing the free body diagram of point B
Let the tension in the section BC and BF are T1 and T2respectively.
From Lami’s theorem
Clearly: 3T1 = T  fi  T1 = 60 N
T1 T2 T
and for the upper pulley: = =
sin120∞ sin120∞ sin120∞
fi T = T1 = T2 = 10 N .

176. (d)
b b
A y B
q q
l
O
M
P Q

T2 = 4T1 = 240 N
where T2 is the force exerted by the celling on the pulley.
As P and Q fall down, the length l decreases at the rate of U m/s.
172. (a)
Force on the table by the block will be equal to normal force between From the figure, l 2 = b2 + y 2
block and table which is equal to weight of block. Normal force on Differentiating with respect to time
table is in downward direction.
dl db dy Ê db dl ˆ
2l ¥ = 2b ¥ + 2y ¥ As = 0, = U ˜
dt dt dt ÁË dt dt ¯
v N
dy Ê l ˆ dl dy Ê 1 ˆ U
fi = ¥ fi = ¥U =
N = mg dt ÁË y ˜¯ dt dt ÁË cosq ˜¯ cosq
173. (b)
177. (c)
N = mg cos q
From the figure for the equilibrium of the system
N
1
2T cosq = 2 mg fi cosq = fi q = 45∞
2
mg sin q
mg cos q T T B
q q
mg
T T
q

174. (d) m m
2m
2W1 cos q = W2
24 Solutions

178. (d) T2 – T1 – 3g = 3a ...(ii)


Force on the pulley by the clamp T1 – g = a ...(iii)

FPC
T
T T
m
60° 60°

mg Mg

Adding (i) and (iii)


−T2 + T1 + 4g = 6a
Fpc = T 2 + [( M + m)g ]2 g
Adding this to (ii) we get g = 9a or a =
9
Fpc = ( Mg )2 + [( M + m)g ]2
T2 T2 T1
2 2
Fpc = M + ( M + m) g

B A C
179. (c) a a a
At 11th second lift is moving upward with acceleration
0 - 3.6
a= = -1.8 m/s2
2 3g T1 5g g
Tension in rope, T = m(g – a)
= 1500 (9.8 – 1.8) = 12000 N 184. (d)
180. (b) From eqn. (iii), T1 = g + a
Acceleration of boy and block will be same and equal to 1.25 m/s2 g 10 g
w.r.t. ground. Hence, or T1 = g + =
9 9
1 185. (d)
10 = (1.25) t 2 fi t = 4 sec
2 From eqn. (i), T2 = 5g – 5a

181. (c) 5g 40 g
= 5g - =
Ê 2m - m ˆ g 9 9
In case-1, the acceleration of the system aa = Á g=
Ë 2 m + m ˜¯ 3
186. (a)
2 mg - mg F.B.D of m1 and m2
In case-2, the acceleration of the system ab = =g
m N T
T
So, aa = ab/3. a
182. (a) a
FBD of mass 2 kg FBD of mass 4 kg m2g
m1g sin 30° m1g cos 30°
T T¢
4N 8N
Equations of motion
2 kg 4 kg Ê 1ˆ
For m1: T - m1g Á ˜ = m1a  ...(i)
20 N 40 N Ë 2¯
T¢ T¢
For m2: m2 g - T = m2 a  ...(ii)
T - T ¢ - 20 = 4  ...(i)
T ¢ - 40 = 8  ...(ii) Ê m1 ˆ
ÁË m2 - 2 ˜¯ g 10
By solving (i) and (ii), T ¢ = 47.23 N and T = 70.8 N From (i) and (ii), a = = m/s2
(m1 + m2 ) 7
183. (c)
1 2
5g T2 = 5a ...(i) Hence distance travelled by block m2 in 2 sec; using s = ut + at
2
Newton's Laws of Motion 25

1 Ê 10 ˆ 20 F + m1g - T2 = m1a
s=0¥2+ ¥ (2)2 = m  2.8 m
2 ÁË 7 ˜¯ 7
fi 1000 + 50 g - T2 = 50 a  ...(iii)
187. (b) T2 – 80 g = 80 a ...(iv)
2mg - mg
Case-I a1 = =g From (iii) and (iv)
m
16000 T 12000 3
(2 m - m) g T2 = N fi 1 = =
Case-II a2 = g= 13 T2 16000 4
(2 m + m) 3
191. (a)
(mg + mg - mg ) g
Case-III a3 = = F = 2T sin q ...(1)
2m 2
T sin q + N = mg
Hence a1 > a3 > a2 fi N = mg – T sin q ...(ii)

188. (c) F F T
T2
T1
q q

T T mg
S1 30° 45° S2
T1 cos 30° T2 cos 45° For no loss of contact:
N>0
5 kg
fi mg – T sin q > 0
T1 cos30∞ = T2 cos 45∞ F
fi mg - > 0 fi F < 2 mg
2
3 1 3 So Fmax = 2 mg
T1 = T2 fi T2 = T1 fi T2 > T1
2 2 2
192. (b)
189. (c) dx dy dx
vx = = .
FBD of M: If M exerts force F = 7Mg/4 on floor, then from third law dt dt dy

floor also exerts force F on box in upward direction.
Since x =  2 - y 2
a
dx y
M \ =-
dy  - y2
2

Mg
F y dy y + | vy |
\ vx = - . =
F – Mg = Ma 2
 -y 2 dt 2 - y2
7 Mg 3g 193. (c)
fi - Mg = Ma fi a =
4 4 For no relative motion between wedge and block, let the acceleration
of both block and wedge be ‘a’ towards left.
190. (c)
In figure-(a): Let tension in the string is T1
F – T1 = m1a
fi 1000 – T1 = 50 a ...(i)
T1 – m2g = m2a
fi T1 – 80 g = 80 a ...(ii)
From FBD of block; N cos q = mg ...(i)
12,000 and F – N sin q = ma ...(ii)
From (i) and (ii), T1 = N
13 From FBD of wedge; N sin q = Ma ...(iii)
In figure-(b): Let tension in the string is T2. from equations (i), (ii) and (iii)
26 Solutions

m 196. (a)
solving we get F = ( M + m) g tan q
M
194. (a)
As the particle moves down the inclined plane, let a be its
acceleration. Now the inclined plane moves horizontally towards
right with acceleration A.

Acceleration of wedge = g sin q


FBD of particle w.r.t. wedge:
Displacement of particle on wedge

Ê h ˆ
x=Á
Ë tan q ˜¯

Considering the vertical component of forces acting on the particle.
Acceleration of particle along displacement = g sin q cos q
mg – R cos q = ma sin q...(i)
For horizontal components the motion of the particle is considered
2s 2h / tan q 2h
at the accelerated frame. So a horizontal fictious force acts opposite t= = =
to A. This pseudo force mA acts towards the left. a g sin q cos q g sin 2 q

197. (c)
The free body diagram of cylinder w.r.t carriage is as shown.
Since net acceleration of cylinder is horizontal,
NAB cos 30° = mg

2
or N AB = mg ...(1)
3

R sin q + mA = ma cos q
R sin q = m(a cos q – A)...(ii)
Now consider the force on inclined plane. The forces are shown in
figure.
R sin q = MA...(iii)
R cos q + Mg = S...(iv)
ma cosq
From equations (ii) and (iii), we get A =
( M + m)
195. (a)
dv ...(i) and NBC = ma + NAB sin 30° ...(2)
bt cos a = m
dt Hence, NAB remains constant and NBC increases with increase in a.
N + bt sin a = mg…(ii)
198. (a)
For breaking off N = 0
The situation is shown in figure.
mg
Therefore from (ii), t = = t0 (say)
b sin a
v t0
From (i), m Ú dv = (b cos a ) Ú t dt
0 0

t0 2 (b cos a ) m 2 g 2
mv = (b cos a ) =
2 2 b2 sin 2 a

mg 2 cos a
v= When the car moves towards right with acceleration a the rope
2b sin 2 a carrying the mass makes an angle q.
Newton's Laws of Motion 27

Forces acting on the body are: g sin q


mg sin q = ma cosq fi a =
(1) Weight mg downwards. cosq
(2) Tension T in the rope. \  Force exerted by the wedge on the block
(3) Pseudo force ma towards left. R = mg cos q + ma sin q
Since the body is in equilibrium under the action of three forces, we
can apply condition of equilibrium. Ê g sin q ˆ mg(cos2 q + sin 2 q )
R = mg cosq + m Á ˜ sin q =
T cos q – mg = 0; T sin q = ma Ë cosq ¯ cosq

a
tan q = or a = g tan q mg
g R=
cosq
1Ê 5 ˆ 10
Acceleration, a = Á 5 - ˜ = m/sec 2 202. (b)
3Ë 3¯ 9
mg cos a
a
10 1 Ê 1ˆ ma a
= 10 tan q or tan q = fi q = tan -1 Á ˜
9 9 Ë 9¯
mg sin a
199. (c) mg a
Acceleration along the plane with respect to lift is, a
a = (a0 + g) sin q
Let the mass of a block is m. It will remain stationary if forces acting
Initial velocity = 0
on it are in equilibrium i.e., ma cos a = mg sin a fi a = g tan a
v2 = u2 + 2as Here ma = Pseudo force on block, mg = Weight.
L
v = 2(a0 + g )sin q 203. (c)
cos q a is acceleration of block w.r.t. wedge.

N
200. (a)
a
mA m
q
45° A
°
a 45
mg cos 45°
in
a g s 45° M
q m

g For block:
N + m A sin 45∞ = mg cos 45∞
When car moves towards right with acceleration a then due to ...(i)
pseudo force the plumb line will tilt in backward direction making For wedge: N sin 45° = MA...(ii)
an angle q with vertical. From (i) and (ii), A = 1 m/s2
From the figure,
tan q = a/g 204. (b)
\ q = tan–1 (a/g) The block (m) will move in vertical direction as seen from ground. If
we observe the block with respect to the wedge (M), its motion will
201. (d) be horizontal towards right.
R ma cos q F.B.D. of m as seen from wedge

q
ma a

mg sin q mA sin q
q
mg cos q + mA
mg ma sin q amM
q
q
When the whole system is accelerated towards left then pseudo mA cos q
force (ma) works on a block towards right.
For the condition of equilibrium N mg
28 Solutions

Equation of motion Hence, from (i)


mA cos q = mamM 
mamM = A cos q    (towards right) vB = 3 iˆ - ˆj

205. (c) Hence, vB = 2 m/s

208. (a)
Tq
1 m/s
ma a a
q 30° q
mg

(Force diagram in the frame of the car)


Applying Newton’s law perpendicular to string l2 l1
mg sin q = ma cos q
a
tan q = q 30°
g 13
m/s 3 m/s
A 4
Applying Newton’s law along string B

fi T - m g 2 + a 2 = ma l1 + l2 = constant

T = m g 2 + a 2 + ma d 1 d  2
fi + =0
dt dt
206. (b)
Let a be the acceleration of cubical block of mass M from ground 13
fi ( 3 cos 30° + 1 cos 60°) + 1 sin q – cos q = 0
frame. Then from the frame of block of mass M, forces on m1 4
and m2 are as shown. Hence, for m1 and m2 to remain at rest in fi 4 sin q – 13 cos q = –8
frame of M
fi 13 cos q – 4 sin q = 8
Ê 3ˆ
m2a m2 Checking the options; we get q = 37° = tan -1 Á ˜ .
Ë 4¯

M m1 209. (a)
m1 g

m1
m2a = m1g or  a = g
m2

m1
\ F= (m1 + m2 + M ) g
m2
Let the magnitude of velocity of top of the rod be vR towards left and
207. (b) the speed of block B be v.
Let AB = l, B = (x, y)
l1 + l2 = constant

vB = vxiˆ + v y ˆj
d 1 d  2
 \ + =0
vB = 3 iˆ + v y ˆj  ...(i) dt dt

x2 + y2 = l2 fi u – vR cos 60° = 0 ...(1)


2x vx = 2y vy = 0 l3 + l4 = constant

y d3 d 4
fi 3+ vy = 0 \ + =0
x dt dt

fi fi vR cos 30° – v = 0 ...(2)


3 + (tan 60∞)v y = 0
fi vy = –1 Solving (1) and (2) v = 3u
Newton's Laws of Motion 29

210. (d) ma cos q > mg sin q


a > g tan q
213. (2.50)
Maximum tension in rope.

T a
Monkey

mg

Equation of motion of monkey.


For block A
T – mg = ma
mg – 2T = ma ...(1)
For block B 250 - 20 ¥ 10 = 20 ¥ a fi a = 2.5 m/s 2

T – mg = mb ...(2)
and by constraint motion 214. (200)
(a – b) + a = 0 ...(3) From combined FBD of man and plank:
Solving (1), (2) and (3) get F
a = –g/5 2F F
and b = – 2g/5
211. (a)
FBD of block A
80 g

F + F + 2F = 80g F = 20 g = 200 N
2m 215. (30)
From frame of car, the effective acceleration (geff) due to gravity
shall be measured as shown in figure. Hence geff makes an angle
2mg 30° with vertical direction (downwards). Since the string aligns
with direction of geff in equilibrium, the required angle is q = 30°.
N cos q = 2mg  ...(i)
[For accelerations of both the block A and wedge B be same, they g
will move along the horizontal]
30° g
N sin q = ma  ...(ii)
30°
2 g  ...(iii)
cot q = geff
a
a = 2g tan q 30°
From (ii)
216. (0.80)
m 2mg
N= ◊ 2 tan q fi N = T = ma (g – a)
sin q cos q
= 0.1 (10 – 2) = 0.8 N

212. (b) 217. (5)


For block to move up In the frame of the lift first pulley will be stationary so velocity of
second pulley will be zero and so in the frame of ground it moves
with 5 m/s.

218. (2)
a = g sin2 q
a = 10 ´ sin2 30°
30 Solutions

1 5 3F
10 ¥ = m/s 2 \ 3T = ma = 3F, a =
 = 4 2 m
1 2 224. (9)
S= at
2 g cosθ
2S 2¥5 g sin q = a g sinθ θ
t= fi t= = 4 =2 s h-r g
a 5/2 cos q = θ (h-r)
r r
219. (0.50)
2aB = (3 – 12t)

Ê dv ˆ 225. (2)
2 Á B ˜ = (3 - 12t )
Ë dt ¯

0
t2
Ú
2 dvB = 3t - 12
2
37°
12
0

0 = t(3 – 6t)
1
fi t= s
2 10

220. (10) N 48 30 18
fi a = 12 cos 37° – 10 sin 37° = - = m/s 2
 N + Ma sin 37° = Mg cos 37° 5 5 5
ma
 N = 10 N
N 1 18 2 36
mg Now, ¥ ¥t = fit=2s
2 5 5
37°
221. (8) 226. (5)
g ˆ gˆ g
m F/2 a1 = j + i , a2 = iˆ
F F 2 4 4
aA = and aB =
2m 4m 227. (6)
F 2m F/2
fi a AB = mg
4m a=
2M + m
1
a AB ¥ t 2 = L
2 Ê mg ˆ
v2 = 2 Á ¥h
2 L ¥ 4m Ë 2 M + m ˜¯
fi t=
F È 0.25 ¥ 10 ˘ -2
= 2Í ˙ ¥ 16.2 ¥ 10
Î 2 + 0.25 ˚
222. (56)
5 ¥ 16.2 ¥ 10-2
Tx1 = 8Tx2 v2 =
0.25
\ xA = 8x2 (Here, x2 is the displacement of load) v = 0.6 m/s

  x2 = 7 m \  xA = 8 ´ 7 = 56 m
228. (0)
dx A
dt Ú
= 3t 2 \ x A = 3t 2 dt
ÊMˆ
ÁË ˜¯ H cosec a g sin a -
M
( H cosec b ) g sin b = a
L L
fi xA = t3 \ t = ( xA ) = (56)
1/ 3 1/ 3
s
fi a = 0

223. (3) x y
H
T α
F=T
T T
3T α β
T T
Newton's Laws of Motion 31

ARCHIVE
1. (b) and speed of bullet just before collision
vbullet = v - gt

v = 100 - 10 = 90 m/s
45° bullet
Now applying conservation of linear momentum just before
F
45° and after the collision

100 N - (0.03)(10) + (0.02)(90) = (0.03 + 0.02) v


100 Þ 5v = 150
From FBD, tan 45° =
F v = 30 m/s
F = 100 N v2
Max. height reached by body h =
2. (a) 2g
dp
= F = kt Before After
dt
0.03 kg v
3P T 10 m/s
ò dP = ò kt dt
P 0 0.05 kg
90 m/s
0.02 kg
kT 2
2 p =
2 30 ´ 20
h= = 45 m
p 2 ´ 10
T =2
k \ Height above tower = 40 m

3. (c) 5. (c)
As spring constant is inversely proportional to the length of Before collision
the spring v1
m
v2
2m

C C
Hence, k1 = and k2 =
l1 l2 After collision
0.5v1 v4
k1 Cl2 l 1 2m m
= l2 = 2 =
k2 l1C nl2 n
Applying linear momentum conservation
4. (c) m1v1iˆ + m2v2iˆ = m1v3iˆ + m2v4iˆ
0.03 kg
m1v1 + 0.5 m1v2 = m1 (0.5 v1 ) + 0.5 m1v4

0.5 m1v1 = 0.5 m1 (v4 - v2 )

v =v -v
100 m 1 4 2
6. (b)
2v v
2m
100 m/s v
m
0.02 kg v=0 45°
m 45°
Time taken for the particles to collide, m
v
d 100 Linear momentum conservation
t = = = 1 sec
vrel 100 v'
m 2v + 2 m v = m ´ 0 + m ´2
Speed of wood just before collision 2
vw = gt = 10 m/s v' = 2 2 v

32 Solutions

7. (a) Force N (2mv)


Man Pressure = =
Area A
Son
v = 104 m/s , m = 10−26 , N = 1022 and A = 1 m2
V 50 kg
20 kg 1022 × 2 × 10−26 × 104
(0.7 – V) P= = 2 N/m 2
1
Option is not matching as per answer, so it is bonus.
9. (a)
Initial linear momentum, Pi = 0
Final linear momentum, Pf = 20(0.7 - V ) - 50(V )
20(0.7 - V ) - 50V = 0

14 − 20V = 50V

14 = 70 V
14
V= = 0.2 m/s
For equilibrium,
70

T sin 45° = F...(1)
8. (*) m
and T cos 45° = 10 g...(2)
But no option is matching v
equation (i)/(ii)
Magnitude of change in momentum per v
collision = 2mv m we get F = 10 g = 100 N

You might also like